Pharm Midterm

Ace your homework & exams now with Quizwiz!

A nurse is preparing to care for a patient who is receiving digoxin. When screening for potential adverse effects from this drug, the nurse will review which of this patient's laboratory results? a. Albumin b. Blood urea nitrogen (BUN) and creatinine c. Hepatic enzymes d. Serum electrolytes

d. Serum electrolytes (Patients with low serum potassium are at risk for fatal cardiac dysrhythmias when taking digoxin, and it is essential to know this level before this medication is administered. Knowing a patient's albumin level would be important when giving drugs that are protein bound. The BUN and creatinine levels are indicators of renal function. Hepatic enzymes are important to know when drugs are metabolized by the liver.)

A child with Prader-Willi syndrome (PWS) has short stature, and the provider is considering treatment with growth hormone. Which aspect of this child's history should the nurse report to the provider? a. Behavior problems b. Low muscle tone c. Mental impairment d. Sleep apnea

d. Sleep apnea

A patient who is receiving a factor VIII concentrate to treat hemophilia A develops hives, a lowgrade temperature elevation, and a stuffy nose. The nurse will prepare to: a. administer subcutaneous epinephrine. b. give an antipyretic medication. c. give oral diphenhydramine. d. provide respiratory support.

c. give oral diphenhydramine. (Factor VIII concentrates can cause allergic reactions, which can range from mild to severe. This patient is showing signs of a mild reaction and can be managed with an antihistamine such as diphenhydramine. Epinephrine is used if there are signs of anaphylaxis, such as wheezing and shortness of breath. Antipyretics are not indicated for low-grade fevers. Respiratory support may be necessary with severe anaphylaxis.)

A patient with chronic renal failure who is on dialysis receives an ESA. The nurse caring for this patient during dialysis notes that the patient's hemoglobin is 10.9 gm/dL, up from the 10.2 gm/dL recorded 2 weeks ago. The patient's blood pressure is 120/80 mm Hg. The nurse will contact the provider to suggest: a. adding an antihypertensive drug. b. discontinuing the ESA. c. giving heparin. d. increasing the dose of the ESA.

c. giving heparin. (ANS: C Patients with CRF on dialysis can benefit from heparin to prevent clotting in the dialysis machine. The risk of clotting is greatest when the hemoglobin level exceeds 11 gm/dL. There is no indication for an antihypertensive drug, because the patient's blood pressure is normal. The patient's hemoglobin is less than 12 gm/dL and has risen less than 1 gm/dL in 2 weeks; therefore, neither reducing the ESA dose nor stopping the ESA is indicated.)

A prescriber orders sumatriptan [Imitrex] for a patient for a migraine headache. Before administration of this drug, it would be most important for the nurse to assess whether the patient: a. has a family history of migraines. b. has taken acetaminophen in the past 3 hours. c. has taken ergotamine in the past 24 hours. d. is allergic to sulfa compounds.

c. has taken ergotamine in the past 24 hours. (Sumatriptan, other triptans, and ergot alkaloids all cause vasoconstriction and should not be combined, or excessive and prolonged vasospasm could result. Sumatriptan should not be used within 24 hours of an ergot derivative and another triptan. A family history is important, but it is not vital assessment data as it relates to this scenario. Acetaminophen has no drug-to-drug interaction with sumatriptan. Sulfa is not a component of sumatriptan and therefore is not relevant.)

A patient who has been prescribed oral ferrous sulfate reports taking extra doses for the past few months. The patient's serum iron level is 560 mcg/dL. What will the nurse expect the provider to order for this patient? a. Discontinuing the ferrous sulfate and rechecking the iron level in 1 month b. Gastric lavage and treatment for acidosis and shock c. Giving oral deferasirox [Exjade] d. Giving parenteral deferoxamine [Desferal]

d. Giving parenteral deferoxamine [Desferal] (If the plasma level of iron is high (above 500 mcg/dL), it should be lowered with parenteral deferoxamine. This level is toxic and must be treated. Gastric lavage is used if unabsorbed tablets are present. Oral deferasirox is used for chronic overload caused by blood transfusions.)

What is the most reliable measure for assessing diabetes control over the preceding 3-month period? a. Self-monitoring blood glucose (SMBG) graph report b. Patient's report c. Fasting blood glucose level d. Glycosylated hemoglobin level

d. Glycosylated hemoglobin level

A patient who has renal impairment will begin taking ranolazine [Ranexa] as an adjunct to nitroglycerine to treat angina. What will the nurse include when teaching this patient? a. "You will need to monitor your blood pressure closely while taking this drug." b. "You should take this drug 1 hour before or 2 hours after a meal." c. "You may experience rapid heart rate while taking this medication." d. "You do not need to worry about drug interactions with this medication."

a. "You will need to monitor your blood pressure closely while taking this drug." (Ranolazine can elevate blood pressure in patients with renal impairment, so patients taking this drug will need to monitor blood pressure. The drug can be taken without regard to food. It does not cause reflex tachycardia. It has many significant drug interactions.)

A patient with arthritis is admitted to the hospital. The patient's serum glucose level is 350 gm/dL, and the blood pressure is 182/98 mm Hg. The nurse notes that the patient's face appears rounded and puffy. The patient complains of feeling weak. What will the nurse do? a. Ask which drugs the patient takes for arthritis. b. Contact the provider to discuss whether the patient has a pituitary carcinoma. c. Request an order for ketoconazole [Nizoral]. d. Suspect that this patient has Addison's disease.

a. Ask which drugs the patient takes for arthritis.

A patient with acromegaly asks the nurse about treatments for this condition. What will the nurse tell the patient? a. Drugs are generally used after surgical and radiation therapies have been tried. b. Drug therapy is easy to administer. c. Drug therapy is inexpensive. d. Drug therapy is generally short term.

a. Drugs are generally used after surgical and radiation therapies have been tried.

Which statement is correct about the contrast between acarbose and miglitol? a. Miglitol has not been associated with hepatic dysfunction. b. With miglitol, sucrose can be used to treat hypoglycemia. c. Miglitol is less effective in African Americans. d. Miglitol has no gastrointestinal side effects.

a. Miglitol has not been associated with hepatic dysfunction.

A patient with hemophilia A is undergoing a tooth extraction at the dentist's office. As an adjunct to factor VIII, what additional medication should the nurse anticipate being ordered for this patient? a. Tranexamic acid [Cyklokapron] b. Acetaminophen [Tylenol] c. Desmopressin [Stimate] d. Vitamin K

a. Tranexamic acid [Cyklokapron] (Antifibrinolytic drugs (eg, aminocaproic acid and tranexamic acid) can be used as adjuncts to factor VIII and factor IX in special situations, such as a tooth extraction. Acetaminophen is indicated for mild pain. Desmopressin is used as replacement therapy but is not used specifically for tooth extractions. Vitamin K is not indicated in this situation.)

A patient who has recently immigrated to the United States from an impoverished country appears malnourished. The patient's folic acid levels are low, and the vitamin B12 levels are normal. The nurse expects this patient's treatment to include: a. a diet high in folic acid. b. intramuscular folic acid. c. oral folic acid and vitamin B12. d. oral folic acid supplements.

a. a diet high in folic acid. (If a folic acid deficiency is caused by poor diet, it should be corrected with dietary measures, not supplements. IM or oral supplements of folic acid are not indicated. Vitamin B12 is not recommended.)

Which is a possible benefit of taking fish-oil supplements? a. A decrease in low-density lipoprotein and triglyceride levels b. Decreased risk of thrombotic stroke c. Prevention of heart disease in high-risk patients d. Reduced risk of dysrhythmia in patients after myocardial infarction

d. Reduced risk of dysrhythmia in patients after myocardial infarction (Fish oil may be beneficial in prevention of heart dysrhythmias in patients who have had myocardial infarction or heart failure. It has not shown to be beneficial in decreasing cholesterol, reducing the risk of thrombotic stroke, or preventing heart disease.)

A patient with a hemoglobin of 7.2 gm/dL begins therapy with an ESA. Two weeks later the patient's hemoglobin is 8.3 gm/dL. The nurse anticipates that the provider will: a. reduce the ESA dose. b. discontinue the ESA. c. order a blood transfusion. d. order heparin.

a. reduce the ESA dose. (The patient's hemoglobin has risen just over 1 gm/dL in 2 weeks, so the dose of ESA should be reduced. The ESA should be discontinued when the hemoglobin is greater than 13 gm/dL. A blood transfusion is not indicated. Heparin is used when patients are undergoing dialysis.)

A patient with a hemoglobin of 7.2 gm/dL begins therapy with an ESA. Two weeks later the patient's hemoglobin is 8.3 gm/dL. The nurse anticipates that the provider will: a. reduce the ESA dose. b. discontinue the ESA. c. order a blood transfusion. d. order heparin.

a. reduce the ESA dose. (The patient's hemoglobin has risen just over 1 gm/dL in 2 weeks, so the dose of ESA should be reduced. The ESA should be discontinued when the hemoglobin is greater than 13 gm/dL. A blood transfusion is not indicated. Heparin is used when patients are undergoing dialysis.)

A nurse is caring for a postoperative patient who has a nasogastric tube with continuous suction. The nurse notes that the patient has shallow respirations and suspects that this patient has developed: a. metabolic acidosis. b. metabolic alkalosis. c. respiratory acidosis. d. respiratory alkalosis.

b. metabolic alkalosis. (Metabolic alkalosis occurs with excessive loss of acid, such as gastric acid, or an excessive increase in alkalinizing salts. The body compensates for metabolic alkalosis by hypoventilating in an attempt to retain CO2. Metabolic acidosis is usually caused by ingestion of acids or excessive loss of bicarbonate and is compensated by hyperventilation. Respiratory acidosis results from hypoventilation. Respiratory alkalosis is the result of hyperventilation.)

A patient is admitted to the hospital and will begin taking levothyroxine [Synthroid]. The nurse learns that the patient also takes warfarin [Coumadin]. The nurse will notify the provider to discuss the dose. a. reducing; levothyroxine b. reducing; warfarin c. increasing; levothyroxine d. increasing; warfarin

b. reducing; warfarin

A patient with hypothyroidism begins taking PO levothyroxine [Synthroid]. The nurse assesses the patient at the beginning of the shift and notes a heart rate of 62 beats/min and a temperature of 97.2°F. The patient is lethargic and difficult to arouse. The nurse will contact the provider to request an order for which drug? a. Beta blocker b. Increased dose of PO levothyroxine c. Intravenous levothyroxine d. Methimazole [Tapazole]

c. Intravenous levothyroxine

What is a desired outcome when a drug is described as easy to administer? a. It can be stored indefinitely without need for refrigeration. b. It does not interact significantly with other medications. c. It enhances patient adherence to the drug regimen. d. It is usually relatively inexpensive to produce.

c. It enhances patient adherence to the drug regimen. (A major benefit of drugs that are easy to administer is that patients taking them are more likely to comply with the drug regimen. Drugs that are easy to give may have the other attributes listed, but those properties are independent of ease of administration.)

Which statement about food and drug interactions is true? a. Foods alter drug absorption and metabolism but not drug action. b. Medications are best absorbed on an empty stomach. c. Patient discomfort is the food and drug interaction of most concern. d. Some foods can inhibit CYP isoenzymes and alter drug metabolism.

d. Some foods can inhibit CYP isoenzymes and alter drug metabolism. (Grapefruit juice inhibits CYP3A4, which lowers the metabolism of some drugs, leading to toxic effects of drugs affected by these isoenzymes. Foods can alter all pharmacokinetic and pharmacodynamic processes. Not all medications are absorbed better on an empty stomach; some require certain foods to enhance absorption. Patient comfort is a concern, but it is not as important as more severe and possibly life-threatening food and drug interactions.)

A patient who has been taking 25 mg of hydrocortisone each morning for several months reports feeling fatigued late in the day each day. What will the nurse tell the patient to discuss with the provider? a. Adding a mineralocorticoid to the drug regimen b. Assessing serum electrolytes to check for toxicity c. Increasing the dose to 50 mg daily d. Splitting the daily dose into a morning and an afternoon dose

d. Splitting the daily dose into a morning and an afternoon dose

An adolescent patient recently attended a health fair and had a serum glucose test. The patient telephones the nurse and says, "My level was 125 mg/dL. Does that mean I have diabetes?" What is the nurse's most accurate response? a. "Unless you were fasting for longer than 8 hours, this does not necessarily mean you have diabetes." b. "At this level, you probably have diabetes. You will need an oral glucose tolerance test this week." c. "This level is conclusive evidence that you have diabetes." d. "This level is conclusive evidence that you do not have diabetes."

a. "Unless you were fasting for longer than 8 hours, this does not necessarily mean you have diabetes."

A nurse is teaching parents how to use an EpiPen for their child, who has a peanut allergy. Which statement by the parents indicates understanding of the teaching? a. "After using the EpiPen, we must go to the emergency department." b. "The EpiPen should be stored in the refrigerator, because epinephrine is sensitive to heat." c. "The teacher should call us when symptoms start so that we can bring the EpiPen to school." d. "We should jab the device into the thigh until it is empty of solution."

a. "After using the EpiPen, we must go to the emergency department." (After using the EpiPen, it is important that the individual get medical attention as quickly as possible. The effects of epinephrine fade in 10 to 20 minutes, and the anaphylactic reaction can be biphasic and prolonged. Epinephrine is sensitive to heat, but storing the device in the refrigerator can compromise the injection mechanism; the device should be stored at room temperature in a dark place. Individuals who need an EpiPen must have the device with them at all times; any delay in treatment can be fatal, because anaphylaxis can occur within minutes after exposure. The EpiPen contains 2 mL of epinephrine, but only 0.3 mL is injected; the device will not be empty with a successful injection.)

A nurse and a nursing student are discussing the plan of care for a patient with schizophrenia. The patient, who has been taking a high-potency FGA for 2 months, has become restless and constantly needs to be in motion. Which statement by the student indicates a need for further education? a. "Anticholinergic medications may help control these symptoms." b. "Because this may be an exacerbation of psychosis, the provider may increase the dose of the FGA." c. "The provider may try a low-potency FGA instead of the high-potency FGA." d. "This patient may need to take a benzodiazepine or a beta blocker."

a. "Anticholinergic medications may help control these symptoms." b. "Because this may be an exacerbation of psychosis, the provider may increase the dose of the FGA." (The patient is showing signs of akathisia, which can resemble an exacerbation of psychosis. If the two are confused and the provider orders more of the FGA, the symptoms may actually increase. Anticholinergic medications may be used, a low-potency FGA may be ordered, or a benzodiazepine or beta blocker may be prescribed.)

A nurse is giving aspirin to a patient during acute management of STEMI. The patient asks why a chewable tablet is given. Which response by the nurse is correct? a. "Aspirin is absorbed more quickly when it is chewed." b. "Chewing aspirin prevents it from being metabolized by the liver." c. "Chewing aspirin prevents stomach irritation." d. "More of the drug is absorbed when aspirin is chewed."

a. "Aspirin is absorbed more quickly when it is chewed." (Aspirin should be chewed to allow rapid absorption across the buccal mucosa. Chewing aspirin does not affect hepatic metabolism, stomach irritation, or the amount absorbed.)

A nurse teaches a nursing student about the differences between desmopressin (DDAVP) and vasopressin [Pitressin]. Which statement by the student indicates a need for further teaching? a. "Desmopressin has a shorter duration of action than vasopressin." b. "Desmopressin is easier to administer than vasopressin." c. "Vasopressin can be used in cardiac resuscitation." d. "Vasopressin can cause serious adverse cardiovascular effects."

a. "Desmopressin has a shorter duration of action than vasopressin."

A patient complains to the nurse that the clonidine [Catapres] recently prescribed for hypertension is causing drowsiness. Which response by the nurse to this concern is appropriate? a. "Drowsiness is a common side effect initially, but it will lessen with time." b. "You may also experience orthostatic hypotension along with the drowsiness." c. "You may be at risk for addiction if you have central nervous system side effects." d. "You should discontinue the medication and contact your prescriber."

a. "Drowsiness is a common side effect initially, but it will lessen with time." (CNS depression, evidenced in this patient by drowsiness, is common in about 35% of the population. These responses become less intense with continued drug use. Orthostatic hypotension is less likely with clonidine, because its antihypertensive effects are not posture dependent. The experience of drowsiness does not indicate abuse potential. The patient should not discontinue the medication abruptly because of the potential for rebound hypertension; the patient should contact the prescriber before stopping the medication.)

A nurse is teaching a group of nursing students about neuropharmacology. Which statement by a student about peripheral nervous system (PNS) drugs indicates a need for further teaching? a. "Drugs affecting axonal conduction have a variety of uses." b. "Drugs that alter synaptic transmission can be highly selective." c. "Many PNS drugs act by altering synaptic transmission." d. "These drugs work by influencing receptor activity."

a. "Drugs affecting axonal conduction have a variety of uses." (Local anesthetics are drugs that work by altering axonal conduction. Any drug affecting axonal conduction would be nonselective, because axonal conduction of impulses is essentially the same in all neurons. In contrast, drugs that affect synaptic transmission can be highly selective, because each transmitter has different effects on receptor sites. Most PNS drugs work on synaptic transmission processes. Through their effects on transmitters, they influence receptor activity.)

A nurse provides teaching for a patient about to begin taking an FGA drug for schizophrenia. Which statement by the patient indicates a need for further teaching about side effects of these drugs? a. "Dry mouth and constipation are uncommon with this medication." b. "I may experience gynecomastia and galactorrhea." c. "I may feel lightheaded or dizzy and should sit or lie down if this occurs." d. "Sedation may occur initially, but will subside in 1 to 2 weeks."

a. "Dry mouth and constipation are uncommon with this medication." (Anticholinergic effects are common with FGAs, so this statement indicates a need for further teaching. Neuroendocrine effects, orthostatic hypertension, and sedation can occur with FGAs.)

A patient recently was diagnosed with bipolar disorder. The patient, who has a history of seasonal allergies, is an athlete who participates in track. The nurse is teaching the patient about lithium [Lithobid], which the prescriber has just ordered. Which statement by the patient indicates the need for further teaching? a. "I can continue to use ibuprofen as needed for muscle pain." b. "I should drink extra fluids before and during exercise." c. "I should not use antihistamines while taking lithium." d. "I should report muscle weakness and tremors to my provider."

a. "I can continue to use ibuprofen as needed for muscle pain." (Because nonsteroidal anti-inflammatory drugs (NSAIDs) can increase lithium levels as much as 60%, they should not be used by patients taking lithium. Aspirin does not have this effect. Lithium induces polyuria in 50% to 70% of patients, so patients should be advised to drink extra fluids, especially during exercise. Antihistamines have anticholinergic effects, which cause urinary hesitancy; this can be uncomfortable when patients experience the polyuria associated with lithium use. Muscle weakness and tremors can occur with lithium; tremors can be treated with beta blockers or by altering the lithium regimen.)

A nurse is teaching a patient who will begin taking verapamil [Calan] for hypertension about the drug's side effects. Which statement by the patient indicates understanding of the teaching? a. "I may become constipated, so I should increase fluids and fiber." b. "I may experience a rapid heart rate as a result of taking this drug." c. "I may have swelling of my hands and feet, but this will subside." d. "I may need to increase my digoxin dose while taking this drug."

a. "I may become constipated, so I should increase fluids and fiber." (Constipation is common with verapamil and can be minimized by increasing dietary fiber and fluids. Verapamil lowers the heart rate. Peripheral edema may occur secondary to vasodilation, and patients should notify their prescriber if this occurs, because the prescriber may use diuretics to treat the condition. Verapamil and digoxin have similar cardiac effects; also, verapamil may increase plasma levels of digoxin by as much as 60%, so digoxin doses may need to be reduced.)

A nurse is teaching a patient who is about to undergo direct-current (DC) cardioversion to treat atrial flutter. The patient has been taking verapamil and warfarin for 6 months. Which statement by the patient indicates understanding of the teaching? a. "I may need long-term therapy with another cardiac medication after the procedure." b. "I should stop taking warfarin a few days before the procedure." c. "I will need to take a beta blocker after the procedure to prevent recurrence of atrial flutter." d. "I will not have to take antidysrhythmic medications after the procedure."

a. "I may need long-term therapy with another cardiac medication after the procedure." (After cardioversion for atrial flutter, patients may continue to need long-term therapy with either a class IC agent or a class III agent to prevent recurrence. Patients undergoing DC cardioversion need to take warfarin 3 to 4 weeks before the procedure and for several weeks afterward. Beta blockers are not indicated for postprocedural prophylaxis. Class IC and class III agents are antidysrhythmic drugs.)

A prescriber has ordered clonidine [Catapres] for a patient who has hypertension. The nurse teaches the patient about the side effects of this drug. Which statement by the patient indicates understanding of the teaching? a. "I should chew sugar-free gum or drink water to reduce dry mouth." b. "I should not drive as long as I am taking this drug." c. "I should stand up slowly when taking this medication." d. "I should stop taking this drug if I feel anxious or depressed."

a. "I should chew sugar-free gum or drink water to reduce dry mouth." (Xerostomia is a common side effect of clonidine and is often uncomfortable enough that patients stop using the drug. Counseling patients to chew sugar-free gum and take frequent sips of liquid can help alleviate this discomfort. Drowsiness is common, but this side effect becomes less intense over time. Patients should be counseled to avoid hazardous activities in the first weeks of therapy if they feel this effect. The hypertensive effects of clonidine are not posture dependent, as they are with the peripheral alpha-adrenergic blockers, so orthostatic hypotension is minimal with this drug. Clonidine (Catapres) causes euphoria, hallucinations, and sedation in high doses and can cause anxiety or depression, although the last two effects are less common. The drug should not be stopped abruptly because of the risk of rebound hypertension, so patients experiencing unpleasant central nervous system (CNS) effects should consult their provider about withdrawing the medication slowly.)

A patient with a new-onset seizure disorder receives a prescription for phenobarbital. The patient reports being concerned about the sedative side effects of this drug. Which response by the nurse is correct? a. "Phenobarbital doses for seizures are nonsedating." b. "This is a short-acting barbiturate, so sedation wears off quickly." c. "Tolerance to the sedative effects will develop in a few weeks." d. "You may actually experience paradoxical effects of euphoria."

a. "Phenobarbital doses for seizures are nonsedating." (Phenobarbital and mephobarbital are used for seizure disorders and suppress seizures at doses that are nonsedative. Phenobarbital is a long-acting barbiturate. At therapeutic doses, sedative effects do not occur. Paradoxical drug effects are associated with benzodiazepines and in older adults and debilitated patients with barbiturates)

A patient who is an opioid addict has undergone detoxification with buprenorphine [Subutex] and has been given a prescription for buprenorphine with naloxone [Suboxone]. The patient asks the nurse why the drug was changed. Which response by the nurse is correct? a. "Suboxone has a lower risk of abuse." b. "Suboxone has a longer half-life." c. "Subutex causes more respiratory depression." d. "Subutex has more buprenorphine."

a. "Suboxone has a lower risk of abuse." (The combination of buprenorphine and naloxone [Suboxone] discourages intravenous abuse, because with IV use, the naloxone precipitates withdrawal; this effect does not occur with sublingual dosing [Subutex]. Suboxone does not differ from Subutex in terms of drug half-life. Subutex does not cause more respiratory depression and does not contain more buprenorphine.)

Lovastatin [Mevacor] is prescribed for a patient for the first time. The nurse should provide the patient with which instruction? a. "Take lovastatin with your evening meal." b. "Take this medicine before breakfast." c. "You may take lovastatin without regard to meals." d. "Take this medicine on an empty stomach."

a. "Take lovastatin with your evening meal." (Patients should be instructed to take lovastatin with the evening meal. Statins should be taken with the evening meal, not before breakfast. Statins should not be administered without regard to meals and should not be taken on an empty stomach.)

A patient with pheochromocytoma is admitted for surgery. The surgeon has ordered an alpha- blocking agent to be given preoperatively. What does the nurse understand about this agent? a. It is ordered to prevent perioperative hypertensive crisis. b. It prevents secretion of catecholamines by the adrenal tumor. c. It reduces contraction of smooth muscles in the adrenal medulla. d. It is given chronically after the surgery to prevent hypertension.

a. . It is ordered to prevent perioperative hypertensive crisis. (Manipulation of the adrenal tumor in patients with pheochromocytoma can cause a massive catecholamine release. Alpha-adrenergic antagonists are given to reduce the risk of acute hypertension during surgery. These agents do not prevent secretion of catecholamines; they block catecholamine receptor sites. They do not act on the tissue of the adrenal medulla. They are given chronically in patients who have inoperable tumors.)

Which patients may receive hydralazine to treat hypertension? (Select all that apply.) a. A 1-month-old infant b. A 5-year-old child c. A pregnant woman d. A mother breast-feeding a newborn e. An older adult

a. A 1-month-old infant b. A 5-year-old child e. An older adult (Hydralazine may be used in infants as young as 1 month of age, in children, and in older adults. Hydralazine is labeled pregnancy category C and data is lacking regarding transmission of hydralazine in breast-feeding women, so benefits should outweigh risks.)

A patient with hypertension is admitted to the hospital. On admission the patient's heart rate is 72 beats/min, and the blood pressure is 140/95 mm Hg. After administering an antihypertensive medication, the nurse notes a heart rate of 85 beats/ min and a blood pressure of 130/80 mm Hg. What does the nurse expect to occur? a. A decrease in the heart rate back to baseline in 1 to 2 days b. An increase in the blood pressure within a few days c. An increase in potassium retention in 1 to 2 days d. A decrease in fluid retention within a week

a. A decrease in the heart rate back to baseline in 1 to 2 days (When blood pressure drops, the baroreceptors in the aortic arch and carotid sinus sense this and relay information to the vasoconstrictor center of the medulla; this causes constriction of arterioles and veins and increased sympathetic impulses to the heart, resulting in an increased heart rate. After 1 to 2 days, this system resets to the new pressure, and the heart rate returns to normal. The blood pressure will not increase when this system resets. Increased potassium retention will not occur. Over time, the body will retain more fluid to increase the blood pressure.)

Patients with a history of myocardial infarction should take which medications indefinitely? (Select all that apply.) a. ACE inhibitors b. Alteplase c. Aspirin d. Beta blockers e. Clopidogrel

a. ACE inhibitors c. Aspirin e. Clopidogrel (Patients who have had an MI should take ACE inhibitors, ASA, and beta blockers indefinitely to prevent recurrence and to minimize continuing cardiac remodeling. Alteplase is given during acute management, and clopidogrel is used during acute management and as an adjunct to reperfusion therapy.)

A prescriber orders clonidine [Kapvay] ER tablets for a 12-year-old child. The nurse understands that this drug is being given to treat which condition? a. ADHD b. Hypertension c. Severe pain d. Tourette's syndrome

a. ADHD (Kapvay ER is used to treat ADHD and is given as a single dose at bedtime. This form of clonidine is not used for hypertension, severe pain, or treatment of Tourette's syndrome.)

Clonidine is approved for the treatment of which conditions? (Select all that apply.) a. ADHD b. Hypertension c. Opioid withdrawal d. Severe pain e. Smoking cessation

a. ADHD b. Hypertension d. Severe pain (Clonidine has three approved uses: treatment of ADHD, hypertension, and severe pain. It has investigational uses for management of opioid withdrawal and for smoking cessation.)

A nurse is caring for a patient who is receiving verapamil [Calan] for hypertension and digoxin [Lanoxin] for heart failure. The nurse will observe this patient for: a. AV blockade. b. gingival hyperplasia. c. migraine headaches. d. reflex tachycardia.

a. AV blockade. (Verapamil and digoxin both suppress impulse conduction through the AV node; when the two drugs are used concurrently, the risk of AV blockade is increased. Gingival hyperplasia can occur in rare cases with verapamil, but it is not an acute symptom. Verapamil can be used to prevent migraine, and its use for this purpose is under investigation. Verapamil and digoxin both suppress the heart rate. Nifedipine causes reflex tachycardia.)

A patient who has migraine headaches is prescribed sumatriptan [Imitrex] 5 mg unit-dose nasal spray. The patient has administered 2 sprays at 1400, 1600, and 1800 and calls to report little relief from headache pain. What will the nurse instruct the patient to do? a. Administer 2 sprays at 2000 and call the provider if no relief. b. Continue using 2 sprays every 2 hours as needed to relieve discomfort. c. Contact the provider to ask about using an ergot alkaloid medication. d. Use 3 sprays at the next dose to increase the dose.

a. Administer 2 sprays at 2000 and call the provider if no relief. (Sumatriptan nasal spray may be used every 2 hours with a maximum 24-hour dose of 40 mg. The patient has already used 30 mg and another 2-spray dose will bring the total dose to 40 mg. The patient should be instructed to contact the provider if the next dose is not effective. Ergot alkaloids should not be given within 24 hours of a triptan. Increasing the next dose to 15 mg would exceed the 24-hour dose maximum.)

The nurse is preparing to administer a dose of epoetin alfa to a patient and notes that the patient has a hemoglobin level of 11.7 gm/dL. Which action by the nurse is correct? a. Administer the dose as ordered. b. Hold the dose and notify the provider. c. Request an order for a reduced dose. d. Suggest that the provider increase the dose.

a. Administer the dose as ordered. (Because the risks of severe cardiovascular events and death are higher in patients whose hemoglobin levels exceed 11 gm/dL, the nurse should hold the dose and notify the provider of this level. It is not correct to administer the dose. Reducing the dose and increasing the dose are not correct because the patient's hemoglobin level is already too high.

A patient has been taking levothyroxine for several years and reports that "for the past 2 weeks, the drug doesn't seem to work as well as before." What will the nurse do? a. Ask the patient when the prescription was last refilled. b. Expect the patient to have an elevated temperature and tachycardia. c. Suggest that the patient begin taking calcium supplements. d. Tell the patient to try taking the medication with food.

a. Ask the patient when the prescription was last refilled.

A patient with renal failure is undergoing chronic hemodialysis. The patient's hemoglobin is 10.6 gm/dL. The provider orders sodium-ferric gluconate complex (SFGC [Ferrlecit]). What will the nurse expect to do? a. Administer the drug intravenously with erythropoietin. b. Give a test dose before each administration of the drug. c. Have epinephrine on hand to treat anaphylaxis if needed. d. Infuse the drug rapidly to achieve maximum effects quickly

a. Administer the drug intravenously with erythropoietin. (SFGC is given parenterally for iron deficiency anemia in patients undergoing chronic hemodialysis. It is always used in conjunction with erythropoietin to stimulate production of red blood cells (RBCs). A test dose is given only with the initial dose and is not necessary with subsequent doses. Anaphylaxis is not a common side effect. The drug should be infused slowly.)

A patient with bipolar disorder has been taking lithium [Lithobid] for several years. The patient has developed a goiter, and serum tests reveal hypothyroidism. What will the nurse expect the provider to order for this patient? a. Administration of levothyroxine b. Increasing the lithium dose c. Iodine supplements d. Referral to an endocrinologist

a. Administration of levothyroxine (Patients taking lithium may experience reduced incorporation of iodine into the thyroid hormone, resulting in goiter and hypothyroidism. Administration of levothyroxine or withdrawing the lithium will reverse both. Increasing the lithium dose will make this worse. Iodine supplements are not indicated. The provider will either order stopping the lithium or administration of levothyroxine, which will reverse this condition, so referral to an endocrinologist is not necessary.)

A child with moderate hemophilia A who weighs 20 kg is admitted to the hospital after a knee injury with bleeding into the joint. The nurse will expect the provider to order which medication? a. Advate 400 units IV over 10 minutes b. Benefix 800 units slow IV push c. Desmopressin 6 mcg IV over 15 to 30 minutes d. Tranexamic acid [Cyklokapron] 200 mg IV

a. Advate 400 units IV over 10 minutes (Advate is a third-generation product for factor VIII replacement, which this child needs. Thirdgeneration products are safer, because they are not exposed to BSA or HAS during production; therefore, this is the product of choice. Benefix is a factor IX replacement product and is not indicated for hemophilia A. Desmopressin is used to stop bleeding in patients with mild hemophilia. Tranexamic acid is used as adjunctive therapy.)

A male patient is being treated for benign prostatic hyperplasia and has stopped taking his alpha- adrenergic antagonist medication because of ejaculatory difficulties. Which medication does the nurse expect the provider to prescribe? a. Alfuzosin [Uroxatral] b. Prazosin [Minipress] c. Silodosin [Rapaflo] d. Tamsulosin [Flomax]

a. Alfuzosin [Uroxatral] (Alfuzosin is used for BPH and does not interfere with ejaculation. All of the other drugs have ejaculatory side effects. Prazosin may be useful for BPH, but it is not approved for this use.)

Which patients are eligible to receive erythropoiesis stimulating agents? (Select all that apply.) a. All patients with nonmyeloid malignancies who require transfusions b. All patients with nonmyeloid malignancies whose chemotherapy is palliative c. All patients whose goal of therapy is curative d. All patients with nonmyeloid malignancies and chemotherapy-induced anemia e. All patients with leukemias or myeloid malignancies

a. All patients with nonmyeloid malignancies who require transfusions b. All patients with nonmyeloid malignancies whose chemotherapy is palliative d. All patients with nonmyeloid malignancies and chemotherapy-induced anemia

Which are medical applications of central nervous system drugs? (Select all that apply.) a. Analgesia b. Anesthesia c. Depression d. Euphoria e. Seizure control

a. Analgesia b. Anesthesia e. Seizure control

A patient is receiving dobutamine [Dobutrex] as a continuous infusion in the immediate postoperative period. The patient also is receiving a diuretic. What adverse drug reactions are possible in this patient? (Select all that apply.) a. Angina b. Dysrhythmias c. Hypotension d. Oliguria e. Tachycardia

a. Angina b. Dysrhythmias e. Tachycardia (Angina, dysrhythmias, and tachycardia are the most common adverse effects of dopamine; general anesthetics can increase the likelihood of dysrhythmias. Dopamine elevates blood pressure by increasing cardiac output. Diuretics complement the beneficial effects of dopamine on the kidney, so urine output would be increased, not decreased.)

Which are therapeutic uses of verapamil? (Select all that apply.) a. Angina of effort b. Cardiac dysrhythmias c. Essential hypertension d. Sick sinus syndrome e. Suppression of preterm labor

a. Angina of effort b. Cardiac dysrhythmias c. Essential hypertension (Verapamil is used to treat both vasospastic angina and angina of effort. It slows the ventricular rate in patients with atrial flutter, atrial fibrillation, and paroxysmal supraventricular tachycardia. It is a first-line drug for the treatment of essential hypertension. It is contraindicated in patients with sick sinus syndrome. Nifedipine has investigational uses in suppressing preterm labor.)

A patient arrives in the emergency department complaining of muscle weakness and drowsiness. The nurse notes a heart rate of 80 beats/min, a respiratory rate of 18 breaths per minute, and a blood pressure of 90/50 mm Hg. The electrocardiogram reveals an abnormal rhythm. The nurse will question the patient about which over-the-counter medication? a. Antacids b. Aspirin c. Laxatives d. Potassium supplements

a. Antacids (Hypermagnesemia can occur when patients are taking magnesium-containing antacids. Symptoms include muscle weakness, sedation, hypotension, and ECG changes. Aspirin would cause metabolic acidosis. Laxatives can contribute to hypokalemia. Potassium supplements would cause hyperkalemia.)

Which drugs will not be affected by interpatient variability? (Select all that apply.) a. Antiseptics applied to the skin to slow bacterial growth b. Antacids to help with the discomfort of heartburn c. Broad-spectrum antibiotics that are effective against many organisms d. Chelating agents that remove metal compounds from the body e. Topical analgesics used to treat localized pain

a. Antiseptics applied to the skin to slow bacterial growth b. Antacids to help with the discomfort of heartburn d. Chelating agents that remove metal compounds from the body (Antiseptics, antacids, and chelating agents are all drugs that do not act through receptors, and therefore that do not depend on the body's processes for effects; these agents react with other molecules. Broad-spectrum antibiotics and topical analgesics bind with receptors to produce desired effects, and these processes can be influenced by individual patient variables.)

A patient arrives in the emergency department complaining of chest pain that has lasted longer than 1 hour and is unrelieved by nitroglycerin. The patient's electrocardiogram reveals elevation of the ST segment. Initial cardiac troponin levels are negative. The patient is receiving oxygen via nasal cannula. Which drug should be given immediately? a. Aspirin 325 mg chewable b. Beta blocker given IV c. Ibuprofen 400 mg orally d. Morphine intravenously

a. Aspirin 325 mg chewable (This patient shows signs of acute ST-elevation myocardial infarction (STEMI). Because cardiac troponin levels usually are not detectable until 2 to 4 hours after the onset of symptoms, treatment should begin as symptoms evolve. Chewable aspirin (ASA) should be given immediately to suppress platelet aggregation and produce an antithrombotic effect. Beta blockers are indicated but do not have to be given immediately. Ibuprofen is contraindicated. Morphine is indicated for pain management and should be administered after aspirin has been given.)

Which two classes of antidysrhythmic drugs have nearly identical cardiac effects? a. Beta blockers and calcium channel blockers b. Beta blockers and potassium channel blockers c. Calcium channel blockers and sodium channel blockers d. Sodium channel blockers and potassium channel blockers

a. Beta blockers and calcium channel blockers (Calcium channel blockade has the same impact on cardiac action potentials as does beta blockade, so these agents have nearly identical effects on cardiac function; that is, they reduce automaticity in the SA node, delay conduction through the AV node, and reduce myocardial contractility. Potassium channel blockers act by delaying repolarization of fast potentials. Sodium channel blockers block sodium channels to slow impulse conduction in the atria, ventricles, and His-Purkinje system.)

A postoperative patient complains of abdominal bloating and discomfort. The nurse caring for this patient will contact the provider to request which medication? a. Bethanechol b. Droperidol c. Promethazine d. Ondansetron

a. Bethanechol (Bethanechol is a muscarinic agonist that is used to treat abdominal distention and urinary retention. The other three agents are antiemetics and would not be useful in this situation.)

What are the indications for administration of a parenteral iron preparation? (Select all that apply.) a. Blood loss of 750 mL/week b. Celiac disease with anemia c. History of alcoholism d. Intestinal disease impairing absorption e. Megaloblastic anemia

a. Blood loss of 750 mL/week b. Celiac disease with anemia e. Megaloblastic anemia (Patients who have lost 750mL/week of iron cannot absorb enough oral iron, so parenteral iron is indicated for these patients. Patients with celiac disease or other diseases that impair iron absorption must receive parenteral iron. Patients with a history of alcoholism are generally deficient in folic acid. Megaloblastic anemia is a symptom of vitamin B12 and/or folic acid deficiency.)

An infant who receives a drug that does not produce CNS side effects in adults exhibits drowsiness and sedation. The nurse understands that this is because of differences in which physiologic system in infants and adults? a. Blood-brain barrier b. First-pass effect c. Gastrointestinal absorption d. Renal filtration

a. Blood-brain barrier (The blood-brain barrier is not fully developed at birth, making infants much more sensitive to CNS drugs than older children and adults. CNS symptoms may include sedation and drowsiness. The first-past effect and GI absorption affect metabolism and absorption of drugs, and renal filtration affects elimination of drugs, all of which may alter drug levels.)

A prescriber has ordered filgrastim (granulocyte colony-stimulating factor) for a patient undergoing myelosuppressive chemotherapy. The nurse will prepare the patient for which potential side effect? a. Bone pain b. Fatigue c. Headache d. Hemorrhage

a. Bone pain (Filgrastim causes bone pain in about 25% of patients. The pain is dose related and usually mild to moderate. Fatigue, bleeding, and headache are not common side effects of filgrastim.)

A male patient reports decreased libido and the nurse notes galactorrhea during a physical assessment. The nurse will report these findings to the provider and will anticipate an order for which medication? a. Cabergoline b. Conivaptan c. Dopamine d. Prolactin

a. Cabergoline

The nurse is providing multiple medications to a patient whose spouse brings grapefruit juice every morning. The nurse will be concerned about which classes of drugs? (Select all that apply.) a. Calcium channel blockers b. Selective serotonin reuptake inhibitors c. Aminoglycosides d. Beta blockers e. Penicillins

a. Calcium channel blockers b. Selective serotonin reuptake inhibitors (Calcium channel blockers and selective serotonin reuptake inhibitors have been shown to reach increased and/or toxic levels when taken with grapefruit juice. Grapefruit juice is not contraindicated with aminoglycosides, beta blockers, or penicillins.)

Which are conditions that may be treated using beta blockers? (Select all that apply.) a. Cardiac dysrhythmias b. Heart failure c. Hypotension d. Hypothyroidism e. Stage fright

a. Cardiac dysrhythmias b. Heart failure e. Stage fright (Beta blockers are used to treat cardiac dysrhythmias, heart failure, and stage fright. They are also used to treat hypertension and hyperthyroidism.)

A patient will be taking amiodarone [Cordarone]. Which baseline tests are necessary before this medication is started? (Select all that apply.) a. Chest radiograph and pulmonary function tests b. Complete blood count with differential c. Ophthalmologic examination d. Renal function tests e. Thyroid function tests

a. Chest radiograph and pulmonary function tests c. Ophthalmologic examination e. Thyroid function tests (Amiodarone has many potential toxic side effects, including pulmonary toxicity, ophthalmic effects, and thyroid toxicity, so these systems should be evaluated at baseline and periodically while the patient is taking the drug. A complete blood count is not indicated. Renal function tests are not indicated.)

A patient is hospitalized with head trauma after a motor vehicle accident. The nurse caring for the patient notes a marked increase in the output of pale, dilute urine. The nurse suspects which condition? a. Diabetes insipidus b. Diabetes mellitus c. Syndrome of inappropriate antidiuretic hormone secretion (SIADH) d. Water intoxication

a. Diabetes insipidus

Which drugs are used to prevent migraine headaches? (Select all that apply.) a. Divalproex [Depakote] b. Amitriptyline [Elavil] c. Timolol d. Ergotamine [Ergomar] e. Acebutolol

a. Divalproex [Depakote] b. Amitriptyline [Elavil] c. Timolol (Preferred drugs for prophylaxis include propranolol, divalproex, and amitriptyline. Timolol is a beta blocker and can be used instead of propranolol. Acebutolol possesses intrinsic sympathomimetic activity and is not effective for migraine prophylaxis. Ergotamine is used for abortive therapy.)

A nurse is screening a patient being admitted to the hospital. The patient reports being fired for drinking at work. On further questioning, the patient reveals a history of daily alcohol consumption of more than six packs of beer each day, regular morning drinking, and several unsuccessful attempts to stop drinking. The nurse detects alcohol on the patient's breath. The nurse will inform the provider of these findings and request an order for which medication? a. Chlordiazepoxide [Librium] b. Clonidine c. Disulfiram [Antabuse] d. Naltrexone [ReVia]

a. Chlordiazepoxide [Librium] (This patient shows signs of alcohol use disorder, according to the AUDIT Screening Instrument, and has a score of at least 21 points from the information included in this history. A score of 8 or higher for men up to age 60 and a score of 4 or higher for others are positive screening results. The patient recently has consumed alcohol, as evidenced by the detectable smell, although the amount consumed and the time since the last drink cannot be determined. Because this patient is an active alcoholic, the risk of withdrawal symptoms is high; therefore, the patient needs medication to facilitate withdrawal. Benzodiazepines are the safest, most effective medications for this purpose, and those with longer half-lives, including chlordiazepoxide, are preferred. Clonidine is useful as an adjunct to help reduce autonomic symptoms associated with withdrawal. Disulfiram is used to maintain abstinence; its use along with alcohol can produce dangerous symptoms. Naltrexone is used to maintain abstinence by reducing cravings; it does not facilitate withdrawal.)

A patient with hypertension has a previous history of opioid dependence. Which medication would the nurse question? a. Clonidine [Catapres] b. Guanabenz [Wytensin] c. Methyldopa d. Reserpine [Serpasil]

a. Clonidine [Catapres] (Patients who abuse cocaine, opioids, and other such drugs also frequently abuse clonidine, so this agent would not be the best choice for this patient. The other drugs do not share this abuse potential.)

A patient is taking sertraline [Zoloft] for depression, and the provider orders azithromycin [Zithromax] to treat an infection. What will the nurse do? a. Contact the provider to discuss an alternative to azithromycin. b. Request an order for a different antidepressant medication. c. Request an order to reduce the dose of sertraline. d. Withhold the sertraline while giving the azithromycin.

a. Contact the provider to discuss an alternative to azithromycin. (Both sertraline and azithromycin prolong the QT interval, and when taken together, they increase the risk of fatal dysrhythmias. Because the antibiotic is used for a short time, it is correct to consider using a different antibiotic. Reducing the dose of sertraline does not alter the combined effects of two drugs that lengthen the QT interval. Sertraline should not be stopped abruptly, so withholding it during antibiotic therapy is not indicated.)

A prescriber has ordered methyldopa for a female patient with hypertension. The nurse understands that which laboratory tests are important before beginning therapy with this drug? (Select all that apply.) a. Coombs' test b. Hemoglobin and hematocrit (H&H) c. Liver function tests d. Pregnancy test e. Urinalysis

a. Coombs' test b. Hemoglobin and hematocrit (H&H) c. Liver function tests (A positive Coombs' test result occurs in 10% to 20% of patients who take methyldopa chronically. A few of these patients (5%) develop hemolytic anemia. Blood should be drawn for a Coombs' test and an H&H before treatment is started and at intervals during treatment. Because methyldopa is associated with liver disorders, liver function tests should be performed before therapy is started and periodically during treatment. Clonidine, not methyldopa, is contraindicated during pregnancy. A urinalysis is not indicated.)

A patient is admitted with nausea, vomiting, diarrhea, and abdominal pain. The patient appears emaciated and complains of feeling weak. The nurse notes a heart rate of 98 beats/min and a blood pressure of 88/54 mm Hg. The nurse reviews the chart and notes an increased serum potassium level and a decreased serum sodium level. The nurse expects the provider to order which medication initially? a. Cosyntropin [Cortrosyn] b. Dexamethasone c. Fludrocortisone [Florinef] d. Hydrocortisone

a. Cosyntropin [Cortrosyn]

The nurse is caring for a patient who is taking a vasodilator that dilates capacitance vessels. The nurse will expect which effect in this patient? a. Decrease in cardiac work b. Increase in cardiac output c. Increase in tissue perfusion d. Increase in venous return

a. Decrease in cardiac work (Vasodilators that dilate capacitance vessels, or veins, lead to a decrease in venous return to the heart, which reduces preload and the force of ventricular contraction. The resultant effect is a decrease in cardiac work. With a decrease in ventricular contraction, cardiac output is reduced, as is tissue perfusion. Dilation of veins causes a decrease in venous return.)

A nurse is caring for a patient who is receiving a drug that causes constriction of arterioles. The nurse expects to observe which effect from this drug? a. Decreased stroke volume b. Increased stroke volume c. Decreased myocardial contractility d. Increased myocardial contractility

a. Decreased stroke volume (Constriction of arterioles increases the load against which the heart must pump to eject blood. Increased constriction of arterioles would decrease, not increase, the stroke volume of the heart. Myocardial contractility is determined by the sympathetic nervous system, acting through beta1- adrenergic receptors in the myocardium.)

Which are effective ways to help prevent medication errors? (Select all that apply.) a. Developing nonpunitive approaches to track errors b. Focusing on caregivers who make errors c. Helping patients to be active, informed members of the healthcare team d. Naming, blaming, and shaming those who make errors e. Using electronic medical order entry systems

a. Developing nonpunitive approaches to track errors c. Helping patients to be active, informed members of the healthcare team e. Using electronic medical order entry systems (To help prevent medication errors, it is important to create an environment for tracking errors that is nonpunitive so that caregivers can learn from mistakes and work together to change systems appropriately. Helping patients be active, informed members of the healthcare team is a useful tool in this process. Using electronic order entry helps eliminate confusion from poor handwriting and allows built-in systems to warn caregivers about possible overdoses, side effects, and drug interactions; it also helps ensure the right dose at the right time to the right patient. An approach that focuses on those who make mistakes by naming, blaming, and shaming is not productive and often results in personnel who cover up mistakes instead of working to make things better.)

A patient is admitted to the hospital after several days of vomiting and diarrhea. After an initial bolus of isotonic (0.9%) sodium chloride solution, the prescriber orders dextrose 5% in 1/2 normal saline (D5NS) with 20 mEq potassium chloride to infuse at a maintenance rate. What should the nurse review before implementing this order? (Select all that apply.) a. Electrocardiogram b. Arterial blood gas levels c. Serum electrolyte levels d. Serum glucose level e. Urine output

a. Electrocardiogram c. Serum electrolyte levels e. Urine output (Patients receiving potassium should be monitored for cardiovascular toxicity; an ECG before and during administration can help monitor for this adverse effect. Serum electrolyte levels should be reviewed to make sure the patient is not already hyperkalemic. Because potassium is excreted via the kidneys, it is important to determine that renal function is intact. Intravenous potassium should never be given if the patient has not voided. Patients who are hyperkalemic can be treated with sodium bicarbonate to increase pH and insulin to promote uptake of potassium by cells, but it is not necessary to evaluate the blood gas or glucose levels before administering potassium.)

Which are indications for discontinuing growth hormone (GH) therapy in children with documented growth hormone deficiency? (Select all that apply.) a. Epiphyseal closure has begun. b. A satisfactory adult height has been achieved. c. Serum GH levels have been normal for 12 consecutive months. d. Normal serum growth hormone levels occur. e. The child no longer responds to the hormone.

a. Epiphyseal closure has begun. b. A satisfactory adult height has been achieved. e. The child no longer responds to the hormone.

The nurse assesses a newly diagnosed patient for short-term complications of diabetes. What does this assessment include? a. Evaluation for hyperglycemia, hypoglycemia, and ketoacidosis b. Cranial nerve testing for peripheral neuropathy c. Pedal pulse palpation for arterial insufficiency d. Auscultation of the carotids for bruits associated with atherosclerosis

a. Evaluation for hyperglycemia, hypoglycemia, and ketoacidosis

A patient is about to undergo a procedure to harvest hematopoietic stem cells from the bone marrow. Which medication will the nurse anticipate giving before this procedure? a. Filgrastim [Neupogen] b. Pegfilgrastim [Neulasta] c. Oprelvekin [Neumega] d. Sargramostim [Leukine]

a. Filgrastim [Neupogen] (Filgrastim is given before harvesting of hematopoietic stem cells (HSCs), because it increases the number of circulating HSCs. Pegfilgrastim is used only to reduce the incidence of infection in patients undergoing chemotherapy of nonmyeloid malignancies. Oprelvekin is used to minimize thrombocytopenia and to reduce the need for platelet transfusion. Sargramostim is used to accelerate myeloid recovery in patients who have undergone autologous bone marrow transplantation.)

What is the target organ when a beta1 agonist is administered? a. Heart b. Kidney c. Respiratory d. Liver

a. Heart (A beta1 agonist increases the patient's heart rate and blood pressure and is used in heart failure. Beta1 agonists would not be used for kidney, respiratory, or liver failure.)

Which two-drug regimen would be appropriate for a patient with hypertension who does not have other compelling conditions? a. Hydrochlorothiazide and nadolol b. Hydralazine and minoxidil c. Spironolactone and amiloride d. Trichlormethiazide and hydrochlorothiazide

a. Hydrochlorothiazide and nadolol (When two or more drugs are used to treat hypertension, each drug should come from a different class. Hydrochlorothiazide is a diuretic and nadolol is a beta blocker, so this choice is appropriate. Hydralazine and minoxidil are vasodilators. Spironolactone and amiloride are potassium-sparing diuretics. Trichlormethiazide and hydrochlorothiazide are both thiazide diuretics.)

A patient has three separate blood pressure (BP) readings of 120/100, 138/92, and 126/96 mm Hg. Which category describes this patient's BP? a. Hypertension b. Isolated systolic hypertension c. Normal d. Prehypertension

a. Hypertension (Hypertension is defined as systolic BP over 140 mm Hg or diastolic BP over 90 mm Hg. When systolic and diastolic BP fall into different categories, classification is based on the higher category. This patient has a hypertensive diastolic BP. Isolated systolic hypertension occurs if the systolic BP is greater than 140 mm Hg with a diastolic BP less than 90 mm Hg. Because this patient has an elevated diastolic BP, it is not considered normal. Prehypertension occurs with a systolic BP of 120 to 139 mm Hg or a diastolic BP of 80 to 89 mm Hg.)

A patient with vitamin B12 deficiency is admitted with symptoms of hypoxia, anemia, numbness of hands and feet, and oral stomatitis. The nurse expects the prescriber to order which of the following therapies? a. IM cyanocobalamin and folic acid b. IM cyanocobalamin and antibiotics c. PO cyanocobalamin and folic acid d. PO cyanocobalamin and blood transfusions

a. IM cyanocobalamin and folic acid (This patient is showing signs of more severe vitamin B12 deficiency with neurologic symptoms; therefore, cyanocobalamin should be given parenterally along with folic acid. Antibiotics are indicated only when signs of infection are present. Oral cyanocobalamin is not recommended.)

A nurse is preparing to administer a medication and learns that it is a nonselective agonist drug. What does the nurse understand about this drug? a. It directly activates receptors to affect many physiologic processes. b. It directly activates receptors to affect a specific physiologic process. c. It prevents receptor activation to affect many physiologic processes. d. It prevents receptor activation to affect a specific physiologic process.

a. It directly activates receptors to affect many physiologic processes. (Drugs that directly activate receptors are called agonists, so this drug will directly activate the receptor site. Drugs that are nonselective activate a variety of receptor sites. A selective agonist would directly activate specific receptors to affect a specific process. An antagonist would prevent receptor activation.)

The nurse is conducting a nursing education class on hemophilia. The nurse correctly states that which characteristics are most likely to be seen in a patient with hemophilia? (Select all that apply.) a. Male gender b. Female gender c. Mother is a carrier. d. All races and ethnicities are susceptible. e. Both parents are carriers.

a. Male gender c. Mother is a carrier. d. All races and ethnicities are susceptible. (Hemophilia is more common in males. The female carries this sex-linked recessive gene. The risk of acquiring hemophilia is shared by all races and ethnic groups. Females typically are carriers but do not have hemophilia. Both parents do not have to carry the gene for their children to inherit the disease.)

In a discussion of drug-drug interactions, which would be the best example of a beneficial inhibitory interaction? a. Naloxone [Narcan] blocking morphine sulfate's actions b. Antacids blocking the action of tetracycline [Sumycin] c. Propanolol [Inderal] blocking the effects of albuterol d. Cholestyramine blocking the actions of antihypertensive drugs

a. Naloxone [Narcan] blocking morphine sulfate's actions (Naloxone is used when a narcotic overdose has occurred. As a narcotic antagonist, it provides a beneficial inhibitory interaction. An antacid blocking tetracycline's antibiotic effects would not be beneficial, but rather detrimental to the desired effects of the tetracycline. Alcohol would not block the effects of opiates, but would contribute to CNS depression. Cholestyramine and certain other adsorbent drugs, which are administered orally but do not undergo absorption, can adsorb other drugs onto themselves, thereby preventing absorption of the other drugs into the blood.)

A patient with lung cancer receives filgrastim [Neupogen] after chemotherapy to reduce neutropenia. The patient reports a moderate degree of bone pain. What will the nurse do? a. Obtain an order to administer acetaminophen. b. Request an order for a complete blood count (CBC). c. Request an order to reduce the dose of filgrastim. d. Suspect metastasis of the cancer to the bone.

a. Obtain an order to administer acetaminophen. (Bone pain occurs in about 25% of patients receiving filgrastim. It can be treated with a nonopioid analgesic and, if necessary, an opioid analgesic. Leukocytosis can occur, but bone pain is not a symptom of this, so a CBC is not indicated. Bone pain is not an indication for reducing the dose of filgrastim. Because moderate bone pain is a common adverse effect, metastasis to the bone is less likely.)

The nurse understands that muscarinic antagonists are often referred by other names. Which names should the nurse recognize for this category of drug? a. Parasympatholytic drugs b. Anticholinergic drugs c. Antimuscarinic drugs d. Muscarinic blockers e. Cholinergic drugs

a. Parasympatholytic drugs b. Anticholinergic drugs c. Antimuscarinic drugs d. Muscarinic blockers (The nurse should recognize that following as being synonymous with muscarinic antagonists: parasympatholytic, anticholinergic, antimuscarinic, and Muscarinic blockers.)

A patient who has been taking clonidine [Catapres] for several weeks complains of drowsiness and constipation. What will the nurse do? a. Recommend that the patient take most of the daily dose at bedtime. b. Suggest asking the provider for a transdermal preparation of the drug. c. Suspect that the patient is overusing the medication. d. Tell the patient to stop taking the drug and call the provider.

a. Recommend that the patient take most of the daily dose at bedtime. (CNS depression is common with clonidine, but this effect lessens over time. Constipation is also a common side effect. Patients who take most of the daily amount at bedtime can minimize daytime sedation. Transdermal forms of clonidine do not alter adverse effects. Patients who are abusing clonidine often experience euphoria and hallucinations along with sedation, but they generally find these effects desirable and would not complain about them to a healthcare provider. Clonidine (Catapres) should not be withdrawn abruptly, because serious rebound hypertension can occur.)

A patient is receiving oral iron for iron deficiency anemia. Which antibiotic drug, taken concurrently with iron, would most concern the nurse? a. Tetracycline b. Cephalosporin c. Metronidazole [Flagyl] d. Penicillin

a. Tetracycline (Coadministration of tetracycline and iron reduces absorption of both iron and tetracycline. Cephalosporin, metronidazole, and penicillin have no significant drug-to-drug interaction with iron.)

The nurse working on a high-acuity medical-surgical unit is prioritizing care for four patients who were just admitted. Which patient should the nurse assess first? a. The NPO patient with a blood glucose level of 80 mg/dL who just received 20 units of 70/30 Novolin insulin b. The patient with a pulse of 58 beats/min who is about to receive digoxin [Lanoxin] c. The patient with a blood pressure of 136/92 mm Hg who complains of having a headache d. The patient with an allergy to penicillin who is receiving an infusion of vancomycin [Vancocin]

a. The NPO patient with a blood glucose level of 80 mg/dL who just received 20 units of 70/30 Novolin insulin

A nurse is reviewing the medications of a patient with diabetes before discharge. The nurse realizes that the patient will be going home on colesevelam, a bile-acid sequestrant, and insulin. What patient education should the nurse provide in the discharge teaching for this patient? a. The patient needs to monitor the blood sugar carefully, because colesevelam can cause hypoglycemia. b. The patient needs to monitor the blood sugar carefully, because colesevelam can cause hyperglycemia. c. The patient needs to take the insulin at least 3 hours before the colesevelam. d. The patient needs to use an oral antidiabetic agent or agents, not insulin, with colesevelam.

a. The patient needs to monitor the blood sugar carefully, because colesevelam can cause hypoglycemia. (Colesevelam can help control hyperglycemia in patients with diabetes; therefore, hypoglycemia is a possible risk. Hyperglycemia is not a risk for patients with diabetes who take colesevelam. Insulin and colesevelam do not interact; therefore, the insulin can be taken at the patient's preferred time or times. Either insulin or oral antidiabetic agents can be taken with colesevelam.)

A patient with type 1 diabetes reports mixing NPH and regular insulin to allow for 1 injection. What should the nurse tell the patient? a. This is an acceptable practice. b. These two forms of insulin are not compatible and cannot be mixed. c. Mixing these two forms of insulin may increase the overall potency of the products. d. NPH insulin should only be mixed with insulin glargine.

a. This is an acceptable practice.

A nursing student asks a nurse about pharmaceutical research and wants to know the purpose of randomization in drug trials. The nurse explains that randomization is used to do what? a. To ensure that differences in outcomes are the result of treatment and not differences in subjects. b. To compare the outcome caused by the treatment to the outcome caused by no treatment c. To make sure that researchers are unaware of which subjects are in which group d. To prevent subjects from knowing which group they are in and prevent preconception bias

a. To ensure that differences in outcomes are the result of treatment and not differences in subjects. (Randomization helps prevent allocation bias, which can occur when researchers place subjects with desired characteristics in the study group and other subjects in the control group so that differences in outcome are actually the result of differences in subjects and not treatment. Comparing treatment outcome to no treatment outcome is the definition of a controlled study. The last two options describe the use of blinding in studies; blinding ensures that researchers or subjects (or both) are unaware of which subjects are in which group so that preconceptions about benefits and risks cannot bias the results.)

A nurse is caring for a child who has hemophilia. While reviewing this child's immunization records, the nurse notes that the child needs the tetanus and reduced diphtheria toxoids and acellular pertussis (Tdap) vaccine and the hepatitis A vaccine. The nurse should notify the provider and obtain an order to give: a. both vaccines intramuscularly. b. both vaccines subcutaneously. c. the hepatitis A vaccine only subcutaneously. d. the Tdap vaccine only intramuscularly.

a. both vaccines intramuscularly. (To minimize the risk of hepatitis, all patients with hemophilia should be fully vaccinated, and all newly diagnosed patients should receive both the hepatitis A and hepatitis B vaccines to minimize the risk of hepatitis from infusions. Clinicians should inject vaccines intramuscularly, taking precautions to prevent excessive bleeding, because subcutaneous administration does not guarantee efficacy. There is no reason to withhold any vaccine in children with hemophilia.)

A patient is taking a vasodilator that relaxes smooth muscles in veins. To help minimize drug side effects, the nurse caring for this patient will: a. caution the patient not to get up abruptly. b. encourage the patient to increase fluid intake. c. tell the patient to report shortness of breath. d. warn the patient about the possibility of bradycardia.

a. caution the patient not to get up abruptly. (Postural hypotension occurs when moving from a supine or seated position to an upright position. It is caused by relaxation of smooth muscle in veins, which allows blood to pool in veins and decreases venous return of blood to the heart. Patients taking such drugs should avoid abrupt transitions to prevent falls. Prolonged use of vasodilators can lead to expansion of blood volume and fluid overload, so increasing fluid intake is not appropriate. Shortness of breath is a symptom associated with heart failure. Tachycardia can occur when the blood pressure drops as a result of the baroreceptor reflex.)

A 1-year-old child with cretinism has been receiving 8 mcg/kg/day of levothyroxine [Synthroid]. The child comes to the clinic for a well-child checkup. The nurse will expect the provider to: a. change the dose of levothyroxine to 6 mcg/kg/day. b. discontinue the drug if the child's physical and mental development are normal. c. increase the dose to accommodate the child's increased growth. d. stop the drug for 4 weeks and check the child's TSH level.

a. change the dose of levothyroxine to 6 mcg/kg/day.

A psychiatric nurse is teaching a patient about an antidepressant medication. The nurse tells the patient that therapeutic effects may not occur for several weeks. The nurse understands that this is likely the result of: a. changes in the brain as a result of prolonged drug exposure. b. direct actions of the drug on specific synaptic functions in the brain. c. slowed drug absorption across the blood-brain barrier. d. tolerance to exposure to the drug over time.

a. changes in the brain as a result of prolonged drug exposure (It is thought that beneficial responses to central nervous system (CNS) drugs are delayed because they result from adaptive changes as the CNS modifies itself in response to prolonged drug exposure, and that the responses are not the result of the direct effects of the drugs on synaptic functions. The blood-brain barrier prevents protein-bound and highly ionized drugs from crossing into the CNS, but it does not slow the effects of drugs that can cross the barrier. Tolerance is a decreased response to a drug after prolonged use.)

A nurse is reviewing a patient's most recent blood count and notes that the patient has a hemoglobin of 9.6 gm/dL and a hematocrit of 33%. The nurse will notify the provider and will expect initial treatment to include: a. determining the cause of the anemia. b. giving intravenous iron dextran. c. giving oral carbonyl iron [Feosol]. d. teaching about dietary iron.

a. determining the cause of the anemia. (Before therapy for iron deficiency anemia is started, the cause must be determined so that the appropriate treatment is given. Oral iron is safer and, most of the time, as effective as parenteral iron, so IV iron is not an initial choice. Oral iron will be given once the cause of the deficiency has been determined. Patients who are iron deficient should be taught about dietary iron, but this is not part of the initial treatment when a deficiency is detected.)

A prescriber orders verapamil [Covera-HS] for a patient who is taking digoxin [Lanoxin] and warfarin. The nurse will expect the prescriber to the dose of . a. lower; digoxin b. increase; digoxin c. lower; warfarin d. increase; warfarin

a. lower; digoxin (Calcium channel blockers, such as verapamil, can increase levels of digoxin, so patients taking these drugs may need to have their digoxin dose reduced. Increasing the dose of digoxin can result in digoxin toxicity. Verapamil does not affect warfarin levels. )

A child who ingested a handful of aspirin tablets from a medicine cabinet at home is brought to the emergency department. The nurse caring for the child notes a respiratory rate of 48 breaths per minute. The nurse understands that this child's respiratory rate is the result of the body's attempt to compensate for: a. metabolic acidosis. b. metabolic alkalosis. c. respiratory acidosis. d. respiratory alkalosis.

a. metabolic acidosis. (Metabolic acidosis can result from the ingestion of aspirin. The body responds by hyperventilating to reduce CO2, which represents volatile carbonic acid, and raise pH. This child has a rapid respiratory rate in response to metabolic acidosis. In patients with metabolic alkalosis, the body responds with hypoventilation in an effort to increase the CO2 level. Patients with respiratory acidosis usually have retention of CO2 secondary to hypoventilation, and compensation is the result of retention of bicarbonate by the kidneys, which is a slow process. Respiratory alkalosis is caused by hyperventilation; treatment involves having the patient rebreathe CO2 or administering sedatives.)

A 5-year-old patient seen in an outpatient clinic is noted to have hypertension on three separate visits. Ambulatory blood pressure monitoring confirms that the child has hypertension. As an initial intervention with the child's parents, the nurse will expect to: a. perform a detailed health history on the child. b. provide teaching about antihypertensive medications. c. reassure the parents that their child may outgrow this condition. d. teach the parents about lifestyle changes and a special diet.

a. perform a detailed health history on the child. (Because the incidence of secondary hypertension is much higher in children than adults, it is important to obtain an accurate health history to help uncover primary causes. Once the type of hypertension is established, the teaching interventions may be useful. Hypertension must be treated, and it is incorrect to reassure parents that their child may just outgrow the condition.)

A nurse is administering a vasodilator that dilates resistance vessels. The nurse understands that this drug will have which effect on the patient? a. Decreased cardiac preload b. Decreased cardiac output c. Increased tissue perfusion d. Increased ventricular contraction

c. Increased tissue perfusion (Vasodilators that dilate resistance vessels, or arterioles, cause a decrease in afterload, which allows cardiac output and tissue perfusion to increase. A decrease in preload would be the result of dilation of capacitance vessels, or veins. Dilation of arterioles increases cardiac output. Ventricular contraction results when preload is increased.)

A nurse is discussing adenosine with a nursing student. Which statement by the student indicates a need for further teaching? a. "Adenosine acts by suppressing action potentials in the SA and AV nodes." b. "Adenosine can be used to prevent paroxysmal supraventricular tachycardia and Wolff-Parkinson-White syndrome." c. "Adenosine has a half-life that lasts only a few seconds and must be given intravenously." d. "Adenosine is not effective for treating atrial fibrillation, atrial flutter, or ventricular dysrhythmias."

b. "Adenosine can be used to prevent paroxysmal supraventricular tachycardia and Wolff-Parkinson-White syndrome." (Adenosine is used to terminate paroxysmal supraventricular tachycardia (SVT) and WolffParkinson-White (WPW) syndrome, not to prevent symptoms. Adenosine suppresses action potentials in the SA and AV nodes. Because it has a very short half-life of 1.5 to 10 seconds, it must be given IV bolus, as close to the heart as possible. Adenosine is not active against atrial fibrillation, atrial flutter, or ventricular dysrhythmias.)

. A nurse provides teaching to a patient with angina who also has type 2 diabetes mellitus, asthma, and hypertension. Which statement by the patient indicates a need for further teaching? a. "An ACE inhibitor, in addition to nitroglycerin, will lower my risk of cardiovascular death." b. "Beta blockers can help me control hypertension." c. "I should begin regular aerobic exercise." d. "Long-acting, slow-release calcium channel blockers can help with anginal pain."

b. "Beta blockers can help me control hypertension." (Beta blockers can be used for angina in most patients but are contraindicated in patients with asthma, because they cause bronchoconstriction. ACE inhibitors help reduce the risk of death in patients with hypertension. Regular aerobic exercise is recommended to control weight and improve cardiovascular function. Long-acting, slow-release CCBs are recommended for patients who have coexisting type 2 diabetes.)

A nurse is explaining activation of beta2 receptors to nursing students during a clinical rotation at the hospital. Which statement by a student demonstrates a need for further teaching? a. "Beta2 activation results in bronchodilation." b. "Beta2 activation results in contraction of uterine muscle." c. "Beta2 activation results in glycogenolysis." d. "Beta2 activation results in vasodilation of skeletal muscles."

b. "Beta2 activation results in contraction of uterine muscle." (Beta2 activation would result in relaxation of uterine smooth muscle, not contraction; this statement indicates a need for further teaching. Bronchodilation is an effect of beta2 activation; no further teaching is needed. Beta2 activation does result in glycogenolysis; no further teaching is needed. Beta2 activation does result in vasodilation of skeletal muscle; no further teaching is needed.)

A nurse is explaining congenital adrenal hyperplasia (CAH) to a group of nursing students. Which statement by a student indicates understanding of the teaching? a. "CAH is caused by a deficiency of ACTH production." b. "CAH is the result of an inability to synthesize glucocorticoids." c. "Newborn screening provides a definitive diagnosis for CAH" d. "The enzyme 21-alpha-hydroxylase increases the production of androgens."

b. "CAH is the result of an inability to synthesize glucocorticoids."

A nurse is teaching a nursing student about the two classes of adrenergic agonist drugs. Which statement by the nursing student indicates understanding of the teaching? a. "Catecholamines may be given orally." b. "Catecholamines often require continuous infusion to be effective." c. "Noncatecholamines do not cross the blood-brain barrier." d. "Noncatecholamines undergo rapid degradation by monoamine oxidase."

b. "Catecholamines often require continuous infusion to be effective." (Catecholamines undergo rapid degradation by monoamine oxidase (MAO) and catechol-O- methyltransferase (COMT). Consequently, they have a brief duration of action, and continuous infusion often is required to maintain the drug's effects. Catecholamines cannot be used orally. Catecholamines do not cross the blood-brain barrier; noncatecholamines do. Noncatecholamines are not degraded by MAO.)

A 12-year-old female patient is admitted to the hospital before sinus surgery. The nurse preparing to care for this patient notes that the admission hemoglobin is 10.2 gm/dL, and the hematocrit is 32%. The nurse will ask the child's parents which question about their daughter? a. "Does she eat green, leafy vegetables?" b. "Has she begun menstruating?" c. "Is she a vegetarian?" d. "Is there a chance she might be pregnant?"

b. "Has she begun menstruating?" (The most common cause of iron deficiency anemia in adolescent females is heavy periods, so asking about menses is an appropriate first question when evaluating the cause of low iron in young females. Iron deficiency only rarely occurs because of poor dietary intake. Although pregnancy is not unheard of in 12-year-old girls, a question about possible pregnancy should not be the first question asked.)

The nurse is providing patient education about glucocorticoid therapy to a patient preparing to be discharged home. Which statement made by the patient best demonstrates understanding of glucocorticoid therapy? a. "I will take the entire dose early with breakfast." b. "I may take two-thirds of the dose in the morning and one-third in the afternoon." c. "I will divide the dose in half and take half in the morning and half in the evening." d. "I will take a dose with each meal."

b. "I may take two-thirds of the dose in the morning and one-third in the afternoon."

A nurse is teaching a patient who will begin taking methimazole [Tapazole] for Graves' disease about the medication. Which statement by the patient indicates understanding of the teaching? a. "Because of the risk for liver toxicity, I will need frequent liver function tests." b. "I should report a sore throat or fever to my provider if either occurs." c. "I will need a complete blood count every few months." d. "It is safe to get pregnant while taking this medication."

b. "I should report a sore throat or fever to my provider if either occurs."

The nurse is providing patient education to a patient who will begin taking fludrocortisone [Florinef] as adjunctive therapy to hydrocortisone. Which statement by the patient indicates understanding of the teaching? a. "I should move from sitting to standing slowly." b. "I should report any swelling of my hands and feet." c. "I should report weight loss to my provider." d. "I should report excessive urine output."

b. "I should report any swelling of my hands and feet."

Which statement is correct regarding the effect of Atropine on the heart? a. Decreases heart rate b. Causes heart block c. Increases heart rate d. Induces cardiac arrest

c. Increases heart rate (Atropine works by causes the heart rate to increase.)

A nurse is reviewing the phenomenon of reflex tachycardia with a group of nursing students. Which statement by a student indicates understanding of this phenomenon? a. "Baroreceptors in the aortic arch stimulate the heart to beat faster." b. "Reflex tachycardia can negate the desired effects of vasodilators." c. "Reflex tachycardia is more likely to occur when beta blockers are given." d. "Venous dilation must occur for reflex tachycardia to occur."

b. "Reflex tachycardia can negate the desired effects of vasodilators." (Reflex tachycardia, which is a compensatory mechanism in response to decreased blood pressure, can negate the desired effect of a vasodilator by eventually increasing blood pressure. Baroreceptors relay information to the vasomotor center of the medulla; the medulla sends impulses to the heart. Beta blockers are given to counter reflex tachycardia. Reflex tachycardia can be produced by dilation of both arterioles and veins.)

A patient who has cancer asks the nurse about using acupuncture to manage cancer pain. What will the nurse tell this patient? a. "Acupuncture is not an effective treatment for cancer pain and should not be used." b. "Studies to date do not clearly indicate effectiveness of acupuncture for alleviating cancer pain." c. "Transcutaneous electrical nerve stimulation (TENS) has been shown to be more effective than acupuncture." d. "There is good evidence to suggest that acupuncture is an effective adjunct treatment for cancer pain."

b. "Studies to date do not clearly indicate effectiveness of acupuncture for alleviating cancer pain." (Studies regarding acupuncture for treatment for cancer pain have been few and not well designed, so there is insufficient evidence to support its use. However, there have not been definitive studies showing that it does not work. TENS has not been well studied, so findings about its use are inconclusive.)

A nurse is teaching a patient who has chronic renal failure and will begin receiving epoetin alfa [Epogen] about this drug therapy. Which statement by the patient indicates understanding of the teaching? a. "If I have to start dialysis, I will have to stop taking this drug." b. "Taking this drug will not eliminate my need for blood transfusions." c. "Taking this medication will prevent the need for dialysis in the future." d. "When I take this, my serum ferritin levels will increase."

b. "Taking this drug will not eliminate my need for blood transfusions." (Epoetin alfa virtually reduces, but does not eliminate, the need for blood transfusions in patients with CRF. Dialysis is not a contraindication for taking this drug. Epoetin alfa does not prevent the need for dialysis. Epoetin alfa does not raise the serum ferritin level.)

. The nurse prepares a patient with Graves' disease for radioactive iodine (131I) therapy. Which statement made by the patient best demonstrates understanding of 131I therapy? a. "I will have to isolate myself from my family for 1 week so that I don't expose them to radiation." b. "This drug will be taken up by the thyroid gland and will destroy the cells to reduce my hyperthyroidism." c. "This drug will help reduce my cold intolerance and weight gain." d. "I will need to take this drug on a daily basis for at least 1 year."

b. "This drug will be taken up by the thyroid gland and will destroy the cells to reduce my hyperthyroidism."

A nurse is giving an enteral medication. The patient asks why this method is preferable for this drug. How will the nurse reply? a. "This route allows more rapid absorption of the drug." b. "This route is safer, less expensive, and more convenient." c. "This route is the best way to control serum drug levels." d. "This route prevents inactivation of the drug by digestive enzymes."

b. "This route is safer, less expensive, and more convenient." (Parenteral routes include the intravenous, intramuscular, and subcutaneous routes. Enteral routes include oral administration, including pills and liquid suspensions. Enteral routes are safer, cheaper, and easier to use. Parenteral routes are used when rapid absorption, precise control of plasma drug levels, and prevention of digestive inactivation are important.)

A patient has begun taking an HMG-COA reductase inhibitor. Which statement about this class of drugs made by the nurse during patient education would be inappropriate? a. "Statins reduce the risk of morbidity from influenza." b. "You should come into the clinic for liver enzymes in 1 month." c. "Statins reduce the risk of coronary events in people with normal LDL levels." d. "You should maintain a healthy lifestyle and avoid high-fat foods."

b. "You should come into the clinic for liver enzymes in 1 month. (Baseline liver enzyme tests should be done before a patient starts taking an HMG-COA reductase inhibitor. They should be measured again in 6 to 12 months unless the patient has poor liver function, in which case the tests are indicated every 3 months. A recent study demonstrated protection against influenza morbidity in patients because of a decrease in proinflammatory cytokine release. Statins do reduce the risk of stroke and coronary events in people with normal LDL levels. Maintaining a healthy lifestyle is important, as is avoiding high-fat foods.)

A patient arrives in the emergency department after becoming dehydrated. Based on the patient's history, the provider determines that isotonic dehydration has occurred. Which solution will the nurse expect to infuse to treat this patient? a. 0.45% sodium chloride in sterile water b. 0.9% sodium chloride in sterile water c. 3% sodium chloride in sterile water d. 5% dextrose solution

b. 0.9% sodium chloride in sterile water (Isotonic dehydration should be treated with an isotonic solution of 0.9% NaCl in sterile water. A 0.45% solution or 5% dextrose is used to treat hypertonic dehydration. A 3% NaCl solution is used to treat hypotonic dehydration.)

A child with hemophilia A who weighs 30 kg receives factor VIII on an ongoing basis as prophylaxis. The provider orders desmopressin [Stimate] to be used for nosebleeds. The nurse will instruct the child's parents to give the medication as nostril as needed. a. 1 spray in each b. 1 spray in one c. 2 sprays in each d. 2 sprays in one

b. 1 spray in one (Children who weigh less than 50 kg should use 1 spray in one nostril for a dose of 150 mcg. Each spray delivers 150 mcg, so 1 spray in both nostrils would give a dose of 300 mcg. Using 2 sprays in one nostril would give a dose of 300 mcg; using 2 sprays in both nostrils would give a dose of 600 mcg.)

A patient tells a nurse that she is thinking about getting pregnant and asks about nutritional supplements. What will the nurse recommend? a. A balanced diet high in green vegetables and grains b. 400 to 800 mg of folic acid per day c. A multivitamin with iron d. Vitamin B12 supplements

b. 400 to 800 mg of folic acid per day (The current recommendation is that all women of child-bearing age receive folic acid supplementation to prevent the development of neural tube defects that can occur early in pregnancy. Dietary folic acid is not sufficient to provide this amount. Iron supplements are given when pregnancy occurs and are not necessary before becoming pregnant. Vitamin B12 supplements are not recommended.)

Two nurses are discussing theories of drug-receptor interaction. Which statements are true regarding the affinity of a drug and its receptor? (Select all that apply.) a. Affinity and intrinsic activity are dependent properties. b. Affinity refers to the strength of the attraction between a drug and its receptor. c. Drugs with high affinity are strongly attracted to their receptors. d. Drugs with low affinity are strongly attracted to their receptors. e. The affinity of a drug for its receptors is reflected in its potency.

b. Affinity refers to the strength of the attraction between a drug and its receptor. c. Drugs with high affinity are strongly attracted to their receptors. e. The affinity of a drug for its receptors is reflected in its potency. (Affinity refers to the strength of the attraction between a drug and its receptor. Drugs with high affinity are strongly attracted to their receptors, and the affinity of a drug and its receptors is reflected in its potency. Affinity and intrinsic activity are independent properties. Drugs with low affinity are weakly attracted to their receptors.)

A nurse is teaching a patient about a prescription for a monoamine oxidase (MAO) inhibitor for depression. What will the nurse teach the patient to avoid while taking this drug? a. Alcoholic beverages b. Aged cheeses c. Brussels sprouts and cabbage d. Grapefruit juice

b. Aged cheeses (Aged cheeses are rich in tyramine, which interacts with MAO inhibitors to raise blood pressure to life-threatening levels. Patients taking MAO inhibitors should be taught to avoid tyramine-rich foods. Chianti wine contains tyramine, but other alcoholic beverages do not. Brussels sprouts and cabbage are foods rich in vitamin K, which can interfere with the effects of warfarin. Grapefruit juice inhibits CYP3A4 and interferes with the metabolism of many medications.)

An infant has allergies and often develops a pruritic rash when exposed to allergens. The infant's parents ask the nurse about using a topical antihistamine. What should the nurse tell them? a. Antihistamines given by this route are not absorbed well in children. b. Applying an antihistamine to the skin can cause toxicity in this age group. c. The child will also need oral medication to achieve effective results. d. Topical medications have fewer side effects than those given by other routes.

b. Applying an antihistamine to the skin can cause toxicity in this age group. (Drug absorption through the skin is more rapid in infants, because their skin is thinner and has greater blood flow; therefore, infants are at increased risk of toxicity from topical drugs. Because of increased drug absorption through the skin, infants should not be given additional drugs via other routes. If a drug is more likely to be absorbed rapidly, it will have more side effects.)

A patient who is receiving reperfusion therapy has a history of heparin-induced thrombosis (HIT). The patient has a creatinine clearance of 28 mL/min. In addition to the fibrinolytic agent, which medication will the nurse expect to administer to this patient? a. Aspirin b. Bivalirudin [Angiomax] c. Clopidogrel [Plavix] d. Fondaparinux [Arixtra]

b. Bivalirudin [Angiomax] (Patients receiving a fibrinolytic medication will also need an anticoagulant to reduce the risk of thrombosis. This patient cannot receive heparin because of the history of HIT, so he or she will need either bivalirudin or fondaparinux. Bivalirudin may be used at reduced doses in patients with a creatinine clearance less than 30 mL/min, but fondaparinux is contraindicated in such patients. Antiplatelet drugs such as aspirin or clopidogrel are not used for this purpose.)

A nurse is educating the staff nurses about ketoacidosis. To evaluate the group's understanding, the nurse asks, "Which sign or symptom would not be consistent with ketoacidosis?" The group gives which correct answer? a. Blood glucose level of 600 mg/dL b. Blood glucose level of 60 mg/dL c. Acidosis d. Ketones in the urine

b. Blood glucose level of 60 mg/dL

Which nonpharmacologic therapies are effective in reducing cancer pain? (Select all that apply.) a. Acupuncture b. Cold c. Exercise d. Heat e. Transcutaneous electrical nerve stimulation (TENS)

b. Cold c. Exercise d. Heat (Cold, heat, and exercise have been shown to reduce pain. Acupuncture and TENS have not been demonstrated to be effective, although in theory they should work.)

A patient who has recurrent migraine headaches is prescribed sumatriptan [Imitrex]. Which aspect of this patient's history is of concern when taking this drug? a. Asthma b. Coronary artery disease c. Diabetes d. Renal disease

b. Coronary artery disease (Serotonin receptor agonists can cause vasoconstriction and coronary vasospasm and should not be given to patients with coronary artery disease, current symptoms of angina, or uncontrolled hypertension. There is no contraindication for asthma, diabetes, or renal disease.)

Which order for potassium (KCl) would the nurse question? (Select all that apply.) a. D5 NS with 20 mEq KCl to start after patient voids b. D5 NS with 60 mEq KCl for a patient with a serum potassium of 3.2 mEq/L c. K-Dur, 1 tablet daily for a patient with diabetic ketoacidosis d. K-Dur, 1 tablet with a full glass of water e. Potassium chloride, 10 mEq rapid IV push

b. D5 NS with 60 mEq KCl for a patient with a serum potassium of 3.2 mEq/L c. K-Dur, 1 tablet daily for a patient with diabetic ketoacidosis e. Potassium chloride, 10 mEq rapid IV push (Patients receiving potassium should not receive potassium in solution diluted at more than 40 mEq/L. Diabetic ketoacidosis causes decreased cellular uptake of potassium, and patients in whom acidosis is present should also receive sodium bicarbonate if they need potassium. IV potassium should not be infused faster than 10 mEq/hr. D5 NS with 20 mEq KCl is an appropriate solution for a patient with intact renal function. Oral potassium should be given with a full glass of water to minimize gastrointestinal (GI) effects.)

What are the effects of prolactin secretion in males? (Select all that apply.) a. Breast development b. Decreased libido c. Delayed puberty d. Galactorrhea e. Infertility

b. Decreased libido c. Delayed puberty d. Galactorrhea

A nurse is preparing to administer a dose of growth hormone and reconstitutes the medication. After adding the diluent, the nurse notices that the preparation is cloudy. What will the nurse do? a. Administer the drug as ordered. b. Discard the drug and prepare another dose. c. Notify the prescriber. d. Shake the drug to dissipate the particles.

b. Discard the drug and prepare another dose.

2. Which medications are used to treat menstrually associated migraine (MAM)? (Select all that apply.) a. Amitriptyline [Elavil] b. Estrogen c. Ergotamine [Ergomar] d. Frovatriptan [Frova] e. Naproxyn

b. Estrogen d. Frovatriptan [Frova] e. Naproxyn (Menstrual migraines may be treated with estrogen, some perimenstrual triptans, such as frovatriptan, and Naproxyn.)

The FDA Amendments Act (FDAAA) was passed in 2007 to address which aspect of drug safety? a. Allowing pharmaceutical companies to identify off-label uses of medications approved for other uses b. Evaluating drug safety information that emerges after a drug has been approved and is in use c. Expediting the approval process of the U.S. Food and Drug Administration (FDA) so that needed drugs can get to market more quickly d. Requiring manufacturers to notify patients before removing a drug from the market

b. Evaluating drug safety information that emerges after a drug has been approved and is in use. (The FDAAA was passed to enable the Food and Drug Administration to continue oversight of a drug after granting it approval so that changes in labeling could be made as necessary and postmarketing risks could be tracked and identified. A provision of the FDA Modernization Act (FDAMA), passed in 1997, allows drug companies to promote their products for off-label uses as long as they promise to conduct studies to support their claims. Regulations to permit accelerated approval of drugs for life-threatening diseases were adopted in 1992 by the FDA. The requirement that drug companies notify patients 6 months before removing a drug from the market is a provision of the FDAMA.)

A nurse counsels a patient with diabetes who is starting therapy with an alpha-glucosidase inhibitor. The patient should be educated about the potential for which adverse reactions? (Select all that apply.) a. Hypoglycemia b. Flatulence c. Elevated iron levels in the blood d. Fluid retention e. Diarrhea

b. Flatulence e. Diarrhea

A patient arrives in the emergency department acutely intoxicated and difficult to arouse. The patient's friends tell the nurse that the patient took a handful of diazepam [Valium] pills while at a party several hours ago. The nurse will expect to administer which drug? a. Buprenorphine [Subutex] b. Flumazenil [Romazicon] c. Nalmefene [Revex] d. Naloxone [Narcan]

b. Flumazenil [Romazicon] (Flumazenil can reverse signs and symptoms of benzodiazepine overdose. Buprenorphine, nalmefene, and naloxone are all used to treat opioid addiction or toxicity.)

Which actions occur in 90% of fatal medication errors? (Select all that apply.) a. Confusing drugs with similar packaging b. Giving a drug intravenously instead of intramuscularly c. Giving Nasarel instead of Nizoral d. Using an infusion device that malfunctions e. Writing a prescription illegibly

b. Giving a drug intravenously instead of intramuscularly c. Giving Nasarel instead of Nizoral e. Writing a prescription illegibly

A patient who is taking a first-generation antipsychotic (FGA) drug for schizophrenia comes to the clinic for evaluation. The nurse observes that the patient has a shuffling gait and mild tremors. The nurse will ask the patient's provider about which course of action? a. Administering a direct dopamine antagonist b. Giving an anticholinergic medication c. Increasing the dose of the antipsychotic drug d. Switching to a second-generation antipsychotic drug

b. Giving an anticholinergic medication (The patient is showing signs of parkinsonism, an extrapyramidal effect associated with antipsychotic medications. Anticholinergic medications are indicated. A direct dopamine antagonist would counter the effects of the antipsychotic and remove any beneficial effect it has. Increasing the dose of the antipsychotic medication would only worsen the extrapyramidal symptoms. A second-generation antipsychotic medication may be used if parkinsonism is severe, since the risk of parkinsonism is lower than with the FGAs. This patient is exhibiting mild symptoms, so this is not necessary at this point.)

A nurse caring for a patient notes that the patient has a temperature of 104°F, and a heart rate of 110 beats/min. The patient's skin is warm and moist, and the patient complains that the room is too warm. The patient appears nervous and has protuberant eyes. The nurse will contact the provider to discuss: a. cretinism. b. Graves' disease. c. myxedema. d. Plummer's disease.

b. Graves' disease.

When administering medications to infants, it is important to remember which of the following? (Select all that apply.) a. Breast-feeding infants are more likely to develop toxicity when given lipid-soluble drugs. b. Immaturity of renal function in infancy causes infants to excrete drugs less efficiently. c. Infants have immature livers, which slow drug metabolism. d. Infants are more sensitive to medications that act on the central nervous system (CNS). e. Oral medications are contraindicated in infants, because PO administration requires a cooperative patient.

b. Immaturity of renal function in infancy causes infants to excrete drugs less efficiently. c. Infants have immature livers, which slow drug metabolism. d. Infants are more sensitive to medications that act on the central nervous system (CNS). (Immature renal function causes infants to excrete drugs more slowly, and infants are at risk for toxicity until renal function is well developed. Infants' livers are not completely developed, and they are less able to metabolize drugs efficiently. Because the blood-brain barrier is not well developed in infants, caution must be used when administering CNS drugs. Lipid-soluble drugs may be excreted in breast milk if the mother is taking them, but breast-feeding does not affect medications given directly to the infant. Oral medications may be given safely to infants as long as they are awake and can swallow the drug.)

Dopamine is administered to a patient who has been experiencing hypotensive episodes. Other than an increase in blood pressure, which indicator would the nurse use to evaluate a successful response? a. Decrease in pulse b. Increase in urine output c. Weight gain d. Improved gastric motility

b. Increase in urine output (Dopamine would cause an increase in urine output, because cardiac output is increased as a result of the increase in blood pressure. The effectiveness of dopamine would not be measured by a decrease in pulse, because dopamine's primary effect is to increase blood pressure. Dopamine's effectiveness would not be evaluated by a weight gain. Dopamine's effectiveness would not be evaluated by improved gastric motility.)

A patient in whom drug therapy has failed several times in the past is readmitted to a hospital to begin therapy for schizophrenia. What will the nurse do to help improve adherence? a. Encourage the patient to take responsibility for medication management. b. Teach the patient about drug side effects and how to manage them. c. Tell the patient that an abstinence syndrome will occur if the drug is stopped. d. Tell the patient that the drug may be taken as needed to control symptoms.

b. Teach the patient about drug side effects and how to manage them. (One way to promote adherence to a medication regimen is to teach patients about drug side effects and how to minimize undesired responses. Family members should be encouraged to oversee medication management for outpatients, because patients themselves may fail to appreciate the need for therapy or may be unwilling to take prescribed medications. It is not true that an abstinence syndrome occurs when these drugs are withdrawn. These drugs are not used PRN; they must be given on a regular basis.)

A patient collapses after running a marathon on a hot day and is brought to the emergency department to be treated for dehydration. The nurse will expect to provide which therapy? a. Intravenous hypertonic fluids given slowly over several hours b. Intravenous hypotonic fluids administered in stages c. Intravenous isotonic fluids given as a rapid bolus d. Oral electrolyte replacement fluids with potassium

b. Intravenous hypotonic fluids administered in stages (Because this patient is experiencing fluid volume loss as the result of excessive sweating, this is most likely hypertonic dehydration, in which loss of water exceeds loss of electrolytes. This should be treated with a hypotonic solution or with fluids that contain no solutes at all. Initial treatment may consist of having the patient drink water. When intravenous therapy is provided, volume replenishment should occur in stages. Hypertonic fluids are used to treat hypotonic contraction, usually caused by excessive sodium loss through the kidneys as the result of diuretic therapy. Isotonic fluids are used to treat isotonic contraction, which is generally caused by vomiting and diarrhea. An oral electrolyte solution would only increase the hypertonicity; if oral rehydration is used in this case, the patient should drink plain water.)

What occurs when a drug binds to a receptor in the body? a. It alters the receptor to become nonresponsive to its usual endogenous molecules. b. It increases or decreases the activity of that receptor. c. It gives the receptor a new function. d. It prevents the action of the receptor by altering its response to chemical mediators.

b. It increases or decreases the activity of that receptor. (When a drug binds to a receptor, it mimics or blocks the actions of the usual endogenous regulatory molecules, either increasing or decreasing the rate of the physiologic activity normally controlled by that receptor. It does not alter the activity of the receptor and does not give the receptor a new function.)

. The nurse is caring for a pregnant patient recently diagnosed with hypothyroidism. The patient tells the nurse she does not want to take medications while she is pregnant. What will the nurse explain to this patient? a. Hypothyroidism is a normal effect of pregnancy and usually is of no consequence. b. Neuropsychologic deficits in the fetus can occur if the condition is not treated. c. No danger to the fetus exists until the third trimester. d. Treatment is required only if the patient is experiencing symptoms.

b. Neuropsychologic deficits in the fetus can occur if the condition is not treated.

A nurse is preparing to administer a drug. Upon reading the medication guide, the nurse notes that the drug has been linked to symptoms of Parkinson disease in some patients. What will the nurse do? a. Ask the patient to report these symptoms, which are known to be teratogenic effects. b. Observe the patient closely for such symptoms and prepare to treat them if needed. c. Request an order to evaluate the patient's genetic predisposition to this effect. d. Warn the patient about these effects and provide reassurance that this is expected.

b. Observe the patient closely for such symptoms and prepare to treat them if needed. (A drug that causes disease-like symptoms is known to be iatrogenic. Nurses should be prepared for this possibility and be prepared to withdraw the drug if necessary and treat the symptoms. Such effects are not teratogenic. Patients with a genetic predisposition to respond differently to drugs are known to have idiosyncratic effects. Iatrogenic effects, even when known, are not typically expected side effects.)

A patient has developed muscarinic antagonist toxicity from ingestion of an unknown chemical. The nurse should prepare to administer which medication? a. Atropine [Sal-Tropine] IV b. Physostigmine [Antilirium] c. An acetylcholinesterase activator d. Pseudoephedrine [Ephedrine]

b. Physostigmine [Antilirium] (Physostigmine is indicated for muscarinic antagonist toxicity. Atropine is a drying agent and would only complicate the drying action that arises from the muscarinic antagonist. An acetylcholinesterase activator would only contribute to dryness that arises from the muscarinic antagonist. Ephedrine is not indicated for muscarinic antagonist toxicity.)

A patient with angina who is taking ranolazine [Ranexa] has developed a respiratory infection and a dysrhythmia. The provider has ordered azithromycin [Zithromax] for the infection and amlodipine for the dysrhythmia. A nursing student caring for this patient tells the nurse that the patient's heart rate is 70 beats/min, and the blood pressure is 128/80 mm Hg. The nurse asks the student to discuss the plan for this patient's care. Which action is correct? a. Observe the patient closely for signs of respiratory toxicity. b. Question the order for azithromycin [Zithromax]. c. Report the patient's increase in blood pressure to the provider. d. Request an order for a different calcium channel blocker.

b. Question the order for azithromycin [Zithromax]. (Agents that inhibit CYP3A4 can increase levels of ranolazine and also the risk of torsades de pointes. Macrolide antibiotics, such as azithromycin, are CYP3A4 inhibitors. Respiratory toxicity is not an expected effect with this patient. The patient's blood pressure is not elevated enough to notify the provider. Amlodipine is the only CCB that should be used with ranolazine)

Which are adverse effects of alpha blockade? (Select all that apply.) a. Hypertension b. Reflex tachycardia c. Nasal congestion d. Ejaculation e. Hypernatremia

b. Reflex tachycardia c. Nasal congestion e. Hypernatremia (Adverse effects of alpha blockade include reflex tachycardia, nasal congestion, and hypernatremia. Other adverse effects include orthostatic hypotension and inhibition of ejaculation. Hypertension and ejaculation are not adverse effects of alpha blockade.)

A nurse recognizes that the actions of benzodiazepines include which findings? (Select all that apply.) a. Sleep deprivation b. Relief of general anxiety c. Suppression of seizures and/or seizure activity d. Development of tardive dyskinesia e. Increase in muscle spasms

b. Relief of general anxiety c. Suppression of seizures and/or seizure activity (Benzodiazepines are indicated to relieve the symptoms of general anxiety and to suppress the central nervous system, thereby suppressing seizures and/or seizure activity. Benzodiazepines cause sleepiness, not sleep deprivation. Benzodiazepines do not cause tardive dyskinesia. Benzodiazepines relax muscles; they do not increase muscle spasms.)

A patient is wheezing and is short of breath. The nurse assesses a heart rate of 88 beats/min, a respiratory rate of 24 breaths per minute, and a blood pressure of 124/78 mm Hg. The prescriber orders a nonspecific beta agonist medication. Besides evaluating the patient for a reduction in respiratory distress, the nurse will monitor for which side effect? a. Hypotension b. Tachycardia c. Tachypnea d. Urinary retention

b. Tachycardia (Beta agonists are used for asthma because of their beta2 effects on bronchial smooth muscle, causing dilation. Beta1 effects cause tachycardia and hypertension. Beta receptors do not exert effects on the bladder.)

A patient has undergone a primary percutaneous coronary intervention with a stent placement. The provider has ordered a daily dose of 81 mg of aspirin and clopidogrel. The patient asks the nurse how long the medications must be taken. What will the nurse tell this patient about the medication regimen? a. This drug regimen will continue indefinitely. b. The clopidogrel will be discontinued in 1 year and the aspirin will be given indefinitely. c. The aspirin will be discontinued in 1 year and the clopidogrel will be given indefinitely. d. Both drugs will be discontinued in 1 year.

b. The clopidogrel will be discontinued in 1 year and the aspirin will be given indefinitely. (Patients who have undergone PCI with a stent will take ASA indefinitely along with an antiplatelet drug for 1 year. The clopidogrel will be discontinued in 1 year, but the aspirin will be given indefinitely.)

A nurse is teaching nursing students about the use of nonproprietary names for drugs. The nurse tells them which fact about nonproprietary names? a. They are approved by the FDA and are easy to remember. b. They are assigned by the U.S. Adopted Names Council. c. They clearly identify the drug's pharmacologic classification. d. They imply the efficacy of the drug and are less complex.

b. They are assigned by the U.S. Adopted Names Council. (Nonproprietary, or generic, names are assigned by the U.S. Adopted Names Council, which ensures that each drug has only one name. Trade names, or brand names, are approved by the FDA and are easier to remember. Some nonproprietary names contain syllables that identify the classification, although not all do. Drug names are not supposed to identify the use for the drug, although some brand names do so.)

A patient newly diagnosed with diabetes expresses concern about losing her vision. Which interventions should be included in the plan of care to reduce this risk? (Select all that apply.) a. Initiation of reliable contraception to prevent pregnancy b. Ways to reduce hyperglycemic episodes c. Use of a prokinetic drug (eg, metoclopramide) d. Smoking cessation e. Emphasis on the importance of taking antihypertensive drugs consistently

b. Ways to reduce hyperglycemic episodes d. Smoking cessation e. Emphasis on the importance of taking antihypertensive drugs consistently

A patient with CRF who will begin receiving an erythropoiesis stimulating agent (ESA) is admitted to the hospital. The nurse notes that the patient's heart rate is 82 beats/min, and the blood pressure is 140/100 mm Hg. A complete blood count reveals a hemoglobin of 8 gm/dL and a hematocrit of 29%. What will the nurse do? a. Administer the ESA and request an order for a blood transfusion. b. Withhold the ESA and request an order for an antihypertensive medication. c. Request an order to increase the dose of ESA to raise the hemoglobin more quickly. d. Suggest initiating dialysis while giving the ESA

b. Withhold the ESA and request an order for an antihypertensive medication. (In patients with CRF, treatment with epoetin alfa is often associated with an increase in blood pressure, so patients with hypertension should be treated before beginning treatment with epoetin alfa. There is no indication for a blood transfusion, an increased dose of ESA, or dialysis.)

A woman with moderate migraine headaches asks a nurse why the provider has ordered metoclopramide [Reglan] as an adjunct to aspirin therapy, because she does not usually experience nausea and vomiting with her migraines. The nurse will tell her that the metoclopramide is used to: a. help induce sleep. b. improve absorption of the aspirin. c. prevent gastric irritation caused by the aspirin. d. prolong the effects of the aspirin.

b. improve absorption of the aspirin. (Besides reducing nausea and vomiting, metoclopramide also reverses gastric stasis and improves absorption of oral antimigraine drugs. When aspirin-like analgesics are combined with metoclopramide, they may work as well as sumatriptan. Metoclopramide is not used to induce sleep. It does not prevent gastric irritation or prolong the effects of the aspirin.)

A nursing student asks a nurse how digoxin causes dysrhythmias. The nurse correctly states that digoxin: a. reduces automaticity in the AV node. b. increases automaticity in the Purkinje fibers. c. increases automaticity in the SA node. d. speeds up AV conduction.

b. increases automaticity in the Purkinje fibers. (Digoxin increases automaticity in the Purkinje fibers, which contributes to dysrhythmias caused by digoxin. Decreased automaticity in the AV node is a desired effect of digoxin. Digoxin does not increase automaticity in the SA node. It does not increase AV node conduction.)

A patient with new-onset exertional angina has taken three nitroglycerin sublingual tablets at 5- minute intervals, but the pain has intensified. The nurse notes that the patient has a heart rate of 76 beats/min and a blood pressure of 120/82 mm Hg. The electrocardiogram is normal. The patient's lips and nail beds are pink, and there is no respiratory distress. The nurse will anticipate providing: a. an angiotensin-converting enzyme (ACE) inhibitor. b. intravenous nitroglycerin and a beta blocker. c. ranolazine (Ranexa) and quinidine. d. supplemental oxygen and intravenous morphine.

b. intravenous nitroglycerin and a beta blocker. (This patient has unstable angina, and the next step, when pain is unrelieved by sublingual nitroglycerin, is to give intravenous nitroglycerin and a beta blocker. ACE inhibitors should be given to patients with persistent hypertension if they have left ventricular dysfunction or congestive heart failure (CHF). Ranolazine is a first-line angina drug, but it should not be given with quinidine because of the risk of increasing the QT interval. Supplemental oxygen is indicated if cyanosis or respiratory distress is present. IV morphine may be given if the pain is unrelieved by nitroglycerin.)

The nurse is taking a medication history on a newly admitted patient. The patient reports taking folic acid and vitamin B12. The nurse notifies the provider because of the concern that folic acid can: a. cause fetal malformation. b. mask the signs of vitamin B12 deficiency. c. negatively affect potassium levels. d. worsen megaloblastic anemia.

b. mask the signs of vitamin B12 deficiency. (Folic acid can reverse the hematologic effects of vitamin B12 deficiency, but it does not reverse the neurologic effects, so it is important to determine the degree of B12 deficiency to treat it. Folic acid does not cause fetal malformation; in fact, it can help prevent neural tube defects. Folic acid does not worsen megaloblastic anemia. Folic acid does not affect potassium levels.)

A nurse obtaining an admission history on an adult patient notes that the patient has a heart rate of 62 beats/min, a blood pressure of 105/62 mm Hg, and a temperature of 96.2°F. The patient appears pale and complains of always feeling cold and tired. The nurse will contact the provider to discuss tests for which condition? a. Cretinism b. Graves' disease c. Hypothyroidism d. Plummer's disease

c. Hypothyroidism

A nurse is caring for a patient after hip replacement surgery. The patient has been receiving iron replacement therapy for 2 days. The nurse notes that the patient's stools appear black. The patient is pale and complains of fatigue. The patient's heart rate is 98 beats/min, respirations are 20 breaths per minute, and the blood pressure is 100/50 mm Hg. The nurse will contact the provider to: a. report possible gastrointestinal hemorrhage. b. request a hemoglobin and hematocrit (H&H). c. request an order for a stool guaiac. d. suggest giving a hypertonic fluid bolus.

b. request a hemoglobin and hematocrit (H&H). (This patient is showing signs of iron deficiency anemia, as manifested by tachycardia and pallor. Because this patient's blood pressure is low, the anemia probably has occurred secondary to blood loss, a common occurrence with hip replacement surgery. The first response should be to obtain an H&H to assess the anemia. GI hemorrhage is not a concern in this patient; black stools are an expected effect of oral iron administration. A stool guaiac is not indicated. If the patient has blood loss that is causing hypotension, an isotonic fluid bolus and packed red blood cells (PRBCs) are indicated to treat this.)

A nurse is teaching nursing students about the use of alpha-adrenergic antagonists. Which statement by a student indicates the need for further teaching? a. "Alpha-adrenergic antagonists block alpha1 receptors on arterioles and veins." b. "Dilation of arterioles has a direct effect on arterial pressure." c. "Dilation of veins by alpha-adrenergic antagonists improves cardiac output." d. "Venous dilation by alpha-adrenergic antagonists indirectly lowers arterial pressure."

c. "Dilation of veins by alpha-adrenergic antagonists improves cardiac output." (Cardiac output is decreased as a result of the venous dilation caused by alpha-adrenergic antagonists. Alpha-adrenergic antagonists block alpha1 receptors on arterioles and veins. When alpha1 receptors on arterioles are blocked by alpha-adrenergic antagonists, a direct effect on arterial pressure occurs. When alpha1 receptors on veins are blocked by alpha-adrenergic antagonists, an indirect effect on arterial pressure occurs.)

A nursing student asks a nurse why patients with chronic renal failure (CRF) have low erythrocyte counts. Which response by the nurse is correct? a. "Damage to the renal tubules increases serum blood loss." b. "Dialysis accelerates the breakdown of red blood cells." c. "Erythropoietin is no longer produced by cells in the kidneys." d. "Patients with CRF are deficient in iron, folic acid, and vitamin B12."

c. "Erythropoietin is no longer produced by cells in the kidneys." (Erythropoietin is normally produced by peritubular cells in the proximal tubules of the kidneys, but it is not produced in patients with CRF. Erythropoietin stimulates red blood cell (RBC) production; therefore, when it is absent, RBC production is compromised. Blood loss in damaged renal tubules does not cause low blood counts in patients with CRF. Dialysis does not contribute to low blood counts. Patients with CRF are not more likely to be deficient in iron, folic acid, and vitamin B12.)

A patient tells the nurse that an analgesic he will begin taking may cause drowsiness and will decrease pain up to 4 hours at a time. Based on this understanding of the drug's effects by the patient, the nurse will anticipate which outcome? a. Decreased chance of having a placebo effect b. Decreased motivation to take the drug c. Improved compliance with the drug regimen d. Increased likelihood of drug overdose

c. Improved compliance with the drug regimen (A drug is effective if it produces the intended effects, even if it also produces side effects. Patients who understand both the risks and benefits of taking a medication are more likely to comply with the drug regimen.)

A nurse is caring for an African American patient with severe hemophilia A who has been admitted for bleeding into the knee joint. The prescriber has ordered intravenous factor VIIa [NovoSeven RT]. A nursing student wants to know why this patient is not receiving factor VIII. Which statement by the nurse is correct? a. "Factor VIIa is stronger than factor VIII and will work faster." b. "Factor VIII is used for prophylaxis, and factor VIIa is used for acute episodes." c. "Factor VIIa is used when patients develop antibodies against factor VIII." d. "Factor VIIa provides immune tolerance therapy so that factor VIII will be more effective."

c. "Factor VIIa is used when patients develop antibodies against factor VIII." (When patients develop antibodies against factor VIII, activated factor VII is used, because it has the same actions as factor VIII. Patients with severe hemophilia A have a 20% to 30% likelihood of developing antibodies to factor VIII and the risk of inhibitor development is unusually high in African American and Hispanic patients, so this patient has an increased risk. Factor VIIa is not stronger than factor VIII. It is not used for prophylaxis only. Immune tolerance therapy describes the use of repeated administration of factor replacement products to eliminate inhibitor production.)

A patient who has type 2 diabetes will begin taking glipizide [Glucotrol]. Which statement by the patient is concerning to the nurse? a. "I will begin by taking this once daily with breakfast." b. "It is safe to drink grapefruit juice while taking this drug." c. "I may continue to have a glass of wine with dinner." d. "I will need to check my blood sugar once daily or more."

c. "I may continue to have a glass of wine with dinner."

A nurse is providing teaching for a nondiabetic adult who develops growth hormone deficiency and who will begin treatment with somatropin [Humatrope]. Which statement by the patient indicates understanding of the teaching? a. "Intramuscular dosing is more effective than subcutaneous dosing." b. "I will have increased muscle mass and strength as well as increased height." c. "I will need to monitor my blood pressure frequently while taking this drug." d. "I will need to take insulin while using this, because it causes hyperglycemia."

c. "I will need to monitor my blood pressure frequently while taking this drug."

A patient who has been taking verapamil [Calan] for hypertension complains of constipation. The patient will begin taking amlodipine [Norvasc] to avoid this side effect. The nurse provides teaching about the difference between the two drugs. Which statement by the patient indicates that further teaching is needed? a. "I can expect dizziness and facial flushing with nifedipine." b. "I should notify the provider if I have swelling of my hands and feet." c. "I will need to take a beta blocker to prevent reflex tachycardia." d. "I will need to take this drug once a day."

c. "I will need to take a beta blocker to prevent reflex tachycardia." (ANS: C Amlodipine produces selective blockade of calcium channels in blood vessels with minimal effects on the heart. Reflex tachycardia is not common, so a beta blocker is not indicated to prevent this effect. Dizziness and facial flushing may occur. Peripheral edema may occur and should be reported to the provider. Amlodipine is given once daily.)

A patient has been taking chlorthalidone to treat hypertension. The patient's prescriber has just ordered the addition of spironolactone to the patient's drug regimen. Which statement by the patient indicates a need for further teaching? a. "I should continue following the DASH diet when adding this drug." b. "I should not take an ACE inhibitor when adding this drug." c. "I will need to take potassium supplements when adding this drug." d. "I will not experience a significant increase in diuresis when adding this drug."

c. "I will need to take potassium supplements when adding this drug." (Spironolactone is given in addition to thiazide diuretics to balance potassium loss caused by the thiazide diuretic. Patients should be advised against taking potassium supplements with spironolactone, because hyperkalemia can result. The DASH diet may be continued. ACE inhibitors are contraindicated because they promote hyperkalemia. Spironolactone does not significantly increase diuresis.)

A patient with bipolar disorder takes lamotrigine [Lamictal]. Which statement by the patient would prompt the nurse to hold the drug and notify the prescriber for further assessment? a. "I get a little dizzy sometimes." b. "I had a headache last week that lasted for about an hour." c. "I've broken out in a rash on my chest and back." d. "Last night I woke up twice with a bad dream."

c. "I've broken out in a rash on my chest and back." (Evidence of a rash in a patient taking lamotrigine requires further assessment, because this may indicate the development of Stevens-Johnson syndrome. Although dizziness and headaches are side effects of lamotrigine, they are not potentially life threatening. A bad dream is not necessarily related to the lamotrigine.)

Parents ask the nurse why an over-the-counter cough suppressant with sedative side effects is not recommended for infants. Which response by the nurse is correct? a. "Babies have a more rapid gastric emptying time and don't absorb drugs well." b. "Cough medicine tastes bad, and infants usually won't take it." c. "Infants are more susceptible to central nervous system effects than are adults." d. "Infants metabolize drugs too rapidly, so drugs aren't as effective."

c. "Infants are more susceptible to central nervous system effects than are adults." (Drugs cross the blood-brain barrier more readily in infants, making these patients more susceptible to central nervous system (CNS) side effects. Infants have a prolonged and irregular gastric emptying time and absorb drugs in the stomach more quickly. Although it may be true that cough medicines taste bad and are difficult to administer, this is not a contraindication to giving them. Infants metabolize drugs more slowly.)

A patient asks a nurse about the effects of chronic alcohol use on the heart. The nurse's best response would be which statement? a. "Chronic alcohol use affects the liver more adversely than it does the heart." b. "Drinking more than two drinks a day protects the heart from atherosclerosis." c. "Long-term alcohol use can damage the heart and cause heart failure." d. "Over time, alcohol use can lower your blood pressure."

c. "Long-term alcohol use can damage the heart and cause heart failure." (Chronic abuse of alcohol results in direct damage to the myocardium, increasing the risk of heart failure. Chronic alcohol abuse has a significant effect on the heart and also affects the liver. Drinking fewer than two alcoholic beverages a day potentially protects the heart from atherosclerosis. Alcohol consumption produces a dose-dependent elevation of blood pressure.)

A nurse is teaching nursing students about the pharmacology of methyldopa. Which statement by a student indicates the need for further teaching? a. "Methyldopa results in alpha2 agonist activation, but it is not itself an alpha2 agonist." b. "Methyldopa is not effective until it is converted to an active compound." c. "Methyldopa reduces blood pressure by reducing cardiac output." d. "Methyldopa's principal mechanism is vasodilation, not cardiosuppression."

c. "Methyldopa reduces blood pressure by reducing cardiac output." (Methyldopa does not reduce the heart rate or cardiac output, so its hypotensive actions are not the result of cardiac depression. The drug is not, in itself, an alpha2 agonist. When taken up into brainstem neurons, it is converted into methylnorepinephrine, which is an alpha2 agonist; it is not effective until converted to this active compound. Its hypotensive effects are the result of vasodilation, not cardiosuppression.)

A nurse is discussing phentolamine [OraVerse] with a nursing student. Which statement by the student indicates the need for further teaching? a. "Phentolamine can be used to block both epinephrine- and norepinephrine- mediated vasoconstriction." b. "Phentolamine can be used to prevent tissue necrosis after extravasation of drugs such as norepinephrine." c. "Phentolamine is a competitive adrenergic agonist that acts selectively on alpha1 receptors." d. "Side effects of phentolamine may include tachycardia and hypotension."

c. "Phentolamine is a competitive adrenergic agonist that acts selectively on alpha1 receptors." (Phentolamine has actions on both alpha1 and alpha2 receptors; it is not selective for alpha1 receptors only. It blocks both epinephrine- and norepinephrine-mediated vasoconstriction. It is used to prevent tissue necrosis after extravasation of drugs such as norepinephrine and other drugs that produce alpha1-mediated vasoconstriction. Side effects include tachycardia and hypotension.)

A patient with schizophrenia receives a dose of risperidone [Risperdal Consta] IM. The nurse teaching this patient about this medication will make which statement? a. "You will experience therapeutic levels of this drug in 1 to 2 weeks." b. "You will need injections of this drug every 6 weeks." c. "You will need to take an oral antipsychotic drug for 3 weeks." d. "You probably will not have extrapyramidal symptoms with this drug."

c. "You will need to take an oral antipsychotic drug for 3 weeks." (Risperidone given intramuscularly is a depot preparation used for long-term therapy. Significant release of the drug does not occur until 2 to 3 weeks after injection; therefore, patients must take an oral antipsychotic medication until drug levels are raised. Therapeutic levels are reached 4 to 6 weeks after injection. Patients need injections every 2 weeks. With IM dosing, the incidence of extrapyramidal symptoms is substantial.)

A patient with type 1 diabetes recently became pregnant. The nurse plans a blood glucose testing schedule for her. What is the recommended monitoring schedule? a. Before each meal and before bed b. In the morning for a fasting level and at 4:00 PM for the peak level c. 6 or 7 times a day d. 3 times a day, along with urine glucose testing

c. 6 or 7 times a day

A patient has allergies and takes an antihistamine. The patient wants to know how the drug works. The nurse understands that antihistamines work because they are what? a. Activators b. Agonists c. Antagonists d. Antidotes

c. Antagonists (Antihistamines bind to receptors to prevent activation by histamine; this makes antihistamines antagonist drugs. Antihistamines do not activate receptors. Agonist drugs activate receptors; they are not antidotes.)

An elderly patient who has type 2 diabetes has a history of severe hypoglycemia. The patient's spouse asks the nurse what the optimum A1c level is for the patient. Which is correct? a. Between 6.5 and 7.0 b. Below 7.0 c. Below 8.0 d. Between 7.0 and 8.5

c. Below 8.0

A pregnant patient is in premature labor. Which class of drug will she be given? a. Alpha1 agonist b. Anticholinergic c. Beta2 agonist d. Beta2 antagonist

c. Beta2 agonist (Beta2 agonists cause relaxation of uterine muscle, slowing or stopping the contractions that precipitate labor. An alpha1 agonist would have effects on the heart and arterioles. Anticholinergic drugs generally are given for their effects on the urinary and GI tracts and do not affect uterine muscle. A beta2 antagonist would cause increased constriction of uterine muscle.)

A patient is diagnosed with anxiety after describing symptoms of tension, poor concentration, and difficulty sleeping that have persisted for over 6 months. Which medication will the nurse expect the provider to order for this patient? a. Alprazolam [Xanax] b. Amitriptyline [Elavil] c. Buspirone [Buspar] d. Paroxetine [Paxil]

c. Buspirone [Buspar] (This patient has symptoms of generalized anxiety disorder (GAD) that are not acute or severe. Buspirone is as effective as benzodiazepines but without causing CNS depression or having the same abuse potential. Symptoms develop slowly, which is acceptable in this case, since symptoms are not acute or severe. Alprazolam is a benzodiazepine and would be used in the short term to treat acute, severe anxiety. Amitriptyline is a TCA used to treat panic disorder. Paroxetine is an antidepressant used as a second-line drug for GAD.)

A patient is given 1 mg of dexamethasone at 11:00 PM; a plasma cortisol level recorded at 8:00 PM the next day is normal. The nurse knows that this is an indication that the patient has what condition? a. Addison's disease b. Congenital adrenal hyperplasia c. Cushing's syndrome d. Secondary adrenal insufficiency

c. Cushing's syndrome

A college student who is unresponsive is brought to the emergency department by friends, who say that their friend drank more than half of a large bottle of whiskey 3 hours ago. Assessment reveals a blood alcohol level of 0.32%. The vital signs are BP, 88/32 mm Hg; R, 6/min; T, 96.8°F; and P, 76/min and weak and thready. The nurse should prepare the patient for which intervention? a. IV fluids and stimulants b. Charcoal administration c. Gastric lavage and dialysis d. Naloxone [Narcan] administration

c. Gastric lavage and dialysis (The average rate at which a person can metabolize alcohol is about 15 mL (0.5 ounce) per hour. The patient in this scenario has consumed more than half of a large bottle of whiskey within 3 hours. Alcohol can be removed from the body by gastric lavage and dialysis. Gastric lavage "washes out" most of the alcohol if any is left in the gut, and dialysis is implemented to reduce the chance of renal failure and cardiovascular shock. Although intravenous fluids may be appropriate, stimulants are contraindicated for this patient. Charcoal is not indicated in this situation. Naloxone is indicated in opiate overdoses, not alcohol overdoses.)

The parents of a boy with hemophilia A want to know why their son will receive factor replacement on an ongoing basis, rather than when needed for bleeding episodes. They tell the nurse that the boy's grandfather, who had the same disease, received the drug only when he had bleeding. The nurse will give the parents which information about ongoing therapy? a. It can reverse progression of the disease. b. It is cheaper, because the amounts of the drug used are less. c. It is used to minimize long-term damage to the joints. d. It prevents the development of inhibitors.

c. It is used to minimize long-term damage to the joints. (Primary prophylaxis, especially in young children, minimizes bleeding episodes and long-term damage to joints. Ongoing treatment does not reverse the progression of the disease. Ongoing therapy is not less expensive, nor does it prevent the development of inhibitors.)

A patient is taking a drug that has known toxic side effects. What will the nurse do? a. Discontinue the drug at the first signs of toxicity. b. Ensure that complete blood counts are ordered periodically. c. Monitor the function of all organs potentially affected by the drug. d. Teach the patient how to treat the symptoms if they develop.

c. Monitor the function of all organs potentially affected by the drug. (When a drug is administered that has known toxic side effects, the nurse is responsible for monitoring all organ systems potentially affected by the drug. Not all toxic side effects warrant discontinuation of the drug, and a nurse cannot discontinue a drug without an order from the provider. Complete blood counts are indicated only for drugs that affect the blood. Some drugs need to be discontinued, so teaching a patient to treat symptoms is not correct in all cases.)

A postoperative patient is reporting pain as a 7 on a scale from 1 to 10, with 10 being the worst pain. The nurse caring for the patient assesses vital signs of HR, 76; RR, 16; and BP, 110/70. The patient has vomited twice. Which postoperative medications will the nurse expect to administer? a. Atropine and morphine b. Bethanechol and ibuprofen c. Morphine and ondansetron [Zofran] d. Promethazine and clonidine [Catapres]

c. Morphine and ondansetron [Zofran] (This patient is experiencing postoperative symptoms of moderate to severe pain and nausea and vomiting. Morphine is used postoperatively for this degree of pain, and ondansetron is one of the most effective antiemetics. Atropine is an anticholinergic drug and usually is given preoperatively or perioperatively to prevent bradycardia. Bethanechol is a muscarinic agonist that is used to counter postoperative abdominal distention and urinary retention. Ibuprofen can be used, but it is effective only for mild postoperative pain. Promethazine is less effective as an antiemetic and can be used, but clonidine, marketed as Catapres, is used for postoperative hypertension.)

A young adult postoperative patient is receiving morphine 2 to 4 mg IV every 2 hours PRN pain. The last dose was 3 mg given 2 hours ago. The patient is asleep, and the nurse notes a heart rate of 86 beats/min and a respiratory rate of 8 breaths per minute. Which PRN medication will the nurse give this patient? a. Diphenhydramine [Benadryl] to counter morphine side effects b. Morphine 4 mg for increased pain, as indicated by tachycardia c. Naloxone [Narcan] to block the effects of the morphine d. Nothing at this time, because the patient is resting comfortably

c. Naloxone [Narcan] to block the effects of the morphine (A respiratory rate of 8 breaths per minute indicates respiratory depression, which is a significant adverse effect indicating morphine toxicity. Naloxone blocks the actions of morphine at cell receptor sites and is given to quickly reverse the effects. This patient does not have signs of an allergic response, which would include shortness of breath, a rapid respiratory rate, and wheezing. The tachycardia might be a sign of worsening pain, but the toxic effects must be treated first. Patients who are sleeping are not always pain free.)

A patient who has migraine headaches has been using sumatriptan [Imitrex] with good results, but reports frequent migraine recurrence 24 hours later. Which medication will the nurse expect the provider to order for this patient? a. Aspirin b. Ergotamine [Ergomar] c. Naratriptan [Amerge] d. Zolmitriptan [Zomig]

c. Naratriptan [Amerge] (Naratriptan has effects that persist longer than other triptans, and the 24-hour recurrence rate may be reduced when taking this formulation. Aspirin has a shorter half-life. Ergotamine and zolmitriptan do not have a long duration.)

Insulin glargine is prescribed for a hospitalized patient who has diabetes. When will the nurse expect to administer this drug? a. Approximately 15 to 30 minutes before each meal b. In the morning and at 4:00 PM c. Once daily at bedtime d. After meals and at bedtime

c. Once daily at bedtime

The nurse is caring for a patient who begins to complain of shortness of breath. The nurse assesses the patient and notes 3+ pitting edema bilaterally in the lower extremities. Which medication taken by the patient causes the most concern? a. Epoetin alfa b. Filgrastim (granulocyte colony-stimulating factor) c. Oprelvekin (interleukin-11) d. Sargramostim (granulocyte-macrophage colony-stimulating factor)

c. Oprelvekin (interleukin-11) (Oprelvekin causes the kidneys to retain sodium and water, which causes peripheral edema and expansion of plasma volume. Some patients experience dyspnea. Fluid retention is not an adverse effect of epoetin alfa, filgrastim, or sargramostim.)

An older adult patient who lives alone and is somewhat forgetful has an overactive bladder (OAB) and reports occasional constipation. The patient has tried behavioral therapy to treat the OAB without adequate results. Which treatment will the nurse anticipate for this patient? a. Oxybutynin short-acting syrup b. Oxybutynin [Ditropan XL] extended-release tablets c. Oxybutynin [Oxytrol] transdermal patch d. Percutaneous tibial nerve stimulation (PTNS)

c. Oxybutynin [Oxytrol] transdermal patch (The transdermal patch is applied weekly and may be the best option for a patient who is more likely to forget to take a daily medication. The transdermal preparation has fewer side effects than the systemic dose, so it is less likely to increase this patient's constipation. The syrup has a high incidence of dry mouth and other anticholinergic side effects. The extended-release tablets must be given daily, and this patient may not remember to take them. PTNS is used after behavioral and drug therapies have failed.)

A prescriber is considering prescribing the amiodarone derivative dronedarone [Multaq] for a patient with atrial flutter. The nurse should be concerned about which of the following? a. History of asthma b. History of hypothyroidism c. PR interval of 260 msec d. QT interval of 520 msec

d. QT interval of 520 msec (Because dronedarone prolongs the QT interval by about 10 msec, it should not be used in patients with a QT interval of more than 500 msec. It does not have significant pulmonary or thyroid toxicity. It should not be used in patients with a PR interval of more than 280 msec.)

A patient with hemophilia is hospitalized for infusion of factor VIII replacement through a venous port. While giving the drug, the nurse notes that the patient's temperature is 101.5°F. The nurse will contact the provider to report which possibility? a. Anaphylactic reaction b. Contamination of factor replacement c. Port infection d. Thrombolytic event

c. Port infection (Central venous access devices are used to facilitate frequent IV administration of factor replacement but pose the risks of infection and thrombosis. This patient has a fever, which is a symptom of infection. Anaphylactic reactions to factor replacement can occur but are characterized by wheezing and shortness of breath, along with swelling in the face. Viral contamination of factor replacement is rare with factor VIII. Thrombolytic events would have symptoms associated with clot formation in certain organs and are not associated with fever.)

The nurse administers naloxone [Narcan] to a patient who has received a toxic dose of morphine sulfate. The nurse understands that the naloxone is effective because of which action? a. Countering the effects of morphine sulfate by agonist actions b. Increasing the excretion of morphine sulfate by altering serum pH c. Preventing activation of opioid receptors through antagonist actions d. Regulating the sensitivity of opioid receptors by neurochemical alterations

c. Preventing activation of opioid receptors through antagonist actions. (Naloxone acts by blocking the action of opioids at opioid receptors. An opioid agonist would increase the effects of morphine. Naloxone does not affect serum pH or excretion of opioids. Naloxone does not alter the sensitivity of opioid receptors.)

A young woman with migraine headaches who has recently begun taking sumatriptan [Imitrex] calls the nurse to report a sensation of chest and arm heaviness. The nurse questions the patient and determines that she feels pressure and not pain. What will the nurse do? a. Ask the patient about any history of hypertension or coronary artery disease. b. Determine whether the patient might be pregnant. c. Reassure the patient that this is a transient, reversible side effect of sumatriptan. d. Tell the patient to stop taking the medication immediately.

c. Reassure the patient that this is a transient, reversible side effect of sumatriptan. (Some patients taking sumatriptan experience unpleasant chest symptoms, usually described as "heavy arms" or "chest pressure." These symptoms are transient and are not related to heart disease. Patients experiencing angina-like pain when taking sumatriptan, as a result of coronary vasospasm, should be asked about hypertension or coronary artery disease (CAD); they should not take sumatriptan if they have a history of either of these. The symptoms this patient describes are not characteristic of pregnancy. There is no need to stop taking the medication.)

A nurse checks a patient's vital signs in the hospital and notes a blood pressure of 146/98 mm Hg. What will the nurse do? a. Instruct the patient to consume a low-sodium diet. b. Prepare the patient for an electrocardiogram and blood tests. c. Recheck the patient's blood pressure in the other arm. d. Request an order for a thiazide diuretic.

c. Recheck the patient's blood pressure in the other arm. (Diagnosis of hypertension should be based on several blood pressure readings, not just one. High readings should be confirmed in the contralateral arm. Low-sodium diets are indicated for patients with confirmed hypertension. An electrocardiogram and blood tests are indicated for patients with confirmed hypertension to rule out primary causes. Thiazide diuretics are first-line drugs for confirmed hypertension.)

A patient has undergone a PCI, and the provider orders clopidogrel to be given for 12 months, along with an ACE inhibitor and heparin. What will the nurse do? a. Question the need for heparin. b. Request an order for a beta blocker. c. Request an order for aspirin. d. Suggest ordering clopidogrel for 14 days.

c. Request an order for aspirin. (Patients who have undergone a PCI should receive heparin, ASA, and a fibrinolytic; therefore, this patient needs ASA added to the drug regimen. Heparin should be given before, during, and for at least 48 to 72 hours after the procedure. Beta blockers are not necessarily indicated. Clopidogrel should be given at least 12 months after the procedure)

A patient in the emergency department is given intravenous diazepam [Valium] for seizures. When the seizures stop, the nurse notes that the patient is lethargic and confused and has a respiratory rate of 10 breaths per minute. The nurse will expect to administer which of the following? a. Flumazenil [Romazicon] b. Gastric lavage c. Respiratory support d. Toxicology testing

c. Respiratory support (When benzodiazepines are administered IV, severe effects, including profound hypotension, respiratory arrest, and cardiac arrest, can occur. Respiration should be monitored, and the airway must be managed if necessary. Flumazenil [Romazicon] is a competitive benzodiazepine receptor antagonist and is used to reverse the sedative effects but may not reverse respiratory depression. Gastric lavage would not be effective, because the benzodiazepine has been given IV. Without further indication of the ingestion of other drugs, toxicology testing is not a priority.)

. A patient with schizophrenia has been taking an oral FGA for 1 week. The patient has been taking the drug daily in two divided doses. The individual complains of daytime drowsiness. The patient's family reports a decrease in the person's hostility and anxiety but states that the patient remains antisocial with disordered thinking. What will the nurse tell the patient and the family? a. An increased dose of the drug may be needed. b. Intramuscular dosing may be needed. c. Some symptoms take months to improve. d. The entire dose may be taken at bedtime.

c. Some symptoms take months to improve. (When patients begin therapy with antipsychotic medications, some symptoms resolve sooner than others. During the first week, agitation, hostility, anxiety, and tension may resolve, but other symptoms may take several months to improve. It is not necessary to increase the dose in the first week. IM dosing is indicated for patients with severe, acute schizophrenia and for long-term maintenance. Sedation is normal, and once an effective dose has been determined, the entire dose can be taken at bedtime, but not in the initial days of therapy.)

A patient reports becoming "immune" to a medication because it no longer works to alleviate symptoms. The nurse recognizes that this decreased effectiveness is likely caused by: a. antagonists produced by the body that compete with the drug for receptor sites. b. decreased selectivity of receptor sites, resulting in a variety of effects. c. desensitization of receptor sites by continual exposure to the drug. d. synthesis of more receptor sites in response to the medication.

c. desensitization of receptor sites by continual exposure to the drug. (Continual exposure to an agonist would cause the cell to become less responsive or desensitized. The body does not produce antagonists as a response to a medication. Receptor site selectivity is determined by physiologic factors and not by the substances that bind to them. Medications do not cause more receptors to be produced.)

A nurse is teaching nursing students about pediatric medication administration. What will the nurse include when discussing pediatric drug research? a. Early studies revealed that less than 10% of drugs known to be effective in adults were effective in children. b. Research findings show that drug doses may be safely calculated by extrapolating adult dosing. c. Studies showed a significant percentage of unanticipated and potentially lethal side effects in children. d. There is no need to continue with pediatric-specific drug research, since early studies were reassuring.

c. Studies showed a significant percentage of unanticipated and potentially lethal side effects in children. (In early studies, about 30% of drugs caused unanticipated side effects, some of them potentially lethal. The same studies revealed that about 20% of drugs were ineffective in children and that about 20% of drugs required doses different from those extrapolated from adult dosing. Because the early studies showed that there is much to learn, the BPCA and PREA were permanently reauthorized by Congress in 2012.)

A patient is being discharged after surgery. During the admission history, the nurse learned that the patient normally consumes 2 or 3 glasses of wine each day. The prescriber has ordered hydrocodone with acetaminophen [Norco] for pain. What will the nurse do? a. Request an order for acetaminophen without hydrocodone for pain. b. Suggest that the patient use ibuprofen for pain. c. Tell the patient not to drink wine while taking Norco. d. Tell the patient to limit wine intake to 1 or 2 glasses per day.

c. Tell the patient not to drink wine while taking Norco. (Combining a hepatotoxic drug with certain other drugs may increase the risk of hepatotoxicity. When even therapeutic doses of acetaminophen are taken with alcohol, the acetaminophen can cause liver damage. Patients should be cautioned not to drink alcohol; even 2 drinks with acetaminophen can produce this effect. Hydrocodone does not contribute to hepatotoxicity. Ibuprofen is not indicated for postoperative pain unless the pain is mild. Limiting wine to 1 or 2 glasses per day still increases the risk of hepatotoxicity.)

A patient in her twenties with Graves' disease who takes methimazole [Tapazole] tells a nurse that she is trying to conceive and asks about disease management during pregnancy. What will the nurse tell her? a. Methimazole is safe to take throughout pregnancy. b. Propylthiouracil should be taken throughout her pregnancy. c. The patient should discuss changing to propylthiouracil from now until her second trimester with her provider. d. The patient should discuss therapy with iodine-131 instead of medications with her provider.

c. The patient should discuss changing to propylthiouracil from now until her second trimester with her provider.

A nursing student is preparing to give a medication that has a boxed warning. The student asks the nurse what this means. What will the nurse explain about boxed warnings? a. They indicate that a drug should not be given except in life-threatening circumstances. b. They provide detailed information about the adverse effects of the drug. c. They alert prescribers to measures to mitigate potential harm from side effects. d. They provide information about antidotes in the event that toxicity occurs.

c. They alert prescribers to measures to mitigate potential harm from side effects. (Boxed warnings (also known as black box warnings) are used to alert providers to potential side effects and to ways to prevent or reduce harm from these side effects. A boxed warning is placed on any drug that, although useful, has serious side effects; this is a way to keep drugs on the market while protecting patients. Many of these drugs are used in situations that are not life threatening. The boxed warning provides a concise summary and not a detailed explanation of drug side effects. The boxed warning does not include antidotes to toxicity.)

A clinic patient who has cirrhosis of the liver develops hypervolemic hypernatremia. Which medication will the nurse expect the provider to order? a. Conivaptan [Vaprisol] b. Desmopressin [DDAVP] c. Tolvaptan [Samsca] d. Vasopressin [Pitressin]

c. Tolvaptan [Samsca]

A patient tells the nurse that the oral drug that has been prescribed has caused a lot of stomach discomfort in the past. What will the nurse ask the prescriber? a. Whether a sublingual form of the medication can be given b. Whether the medication can be given by a parenteral route instead c. Whether an enteric-coated form of the drug is available d. Whether the patient can receive a sustained-release preparation of the drug

c. Whether an enteric-coated form of the drug is available. (Enteric-coated drugs are preparations that have been coated with a material that dissolves in the intestines, not the stomach. This coating is used either to protect the drug from stomach acid and pepsin or to protect the stomach from a drug that can cause gastric upset. Sublingual forms often are used for drugs that undergo rapid inactivation during the first pass through the hepatic circulation so that the drug can be absorbed directly into the systemic circulation. Parenteral routes are more costly and less safe than oral administration and should not be used unless necessary. A sustained-release preparation is used to release the drug into the body over a specific period to reduce the number of daily doses required to sustain therapeutic drug levels.)

A patient has had blood pressures of 150/95 and 148/90 mm Hg on two separate office visits. The patient reports a blood pressure of 145/92 mm Hg taken in an ambulatory setting. The patient's diagnostic tests are all normal. The nurse will expect this patient's provider to order: a. a beta blocker. b. a loop diuretic and spironolactone. c. a thiazide diuretic. d. counseling on lifestyle changes

c. a thiazide diuretic. (This patient has primary, or essential, hypertension as evidenced by systolic pressure greater than 140 and diastolic pressure greater than 90, along with normal tests ruling out another primary cause. Thiazide diuretics are first-line drugs for hypertension. Beta blockers are effective but are most often used to counter reflex tachycardia associated with reduced blood pressure caused by therapeutic agents. Loop diuretics cause greater diuresis than is usually needed and so are not first-line drugs. This patient should be counseled on lifestyle changes as an adjunct to drug therapy but should also begin drug therapy because hypertension already exists.)

A patient who has chronic adrenal insufficiency is admitted to the hospital for an open cholecystectomy. The nurse obtaining the admission history learns that the patient takes hydrocortisone 25 mg PO daily in the morning. The patient's surgery is scheduled for the next morning. The nurse will expect an order to: a. administer the usual morning dose of hydrocortisone 25 mg PO. b. administer hydrocortisone 50 mg PO in the morning. c. administer hydrocortisone 50 mg IV before surgery. d. withhold the morning dose of hydrocortisone and give it after surgery.

c. administer hydrocortisone 50 mg IV before surgery.

A nurse is administering drug X to a patient. The drug information states that the drug acts by activating receptors in the peripheral nervous system by increasing transmitter synthesis. The nurse understands that the effect of this drug is to: a. activate axonal conduction. b. enhance transmitter storage. c. increase receptor activation. d. synthesize supertransmitters.

c. increase receptor activation. (Drugs that increase transmitter synthesis increase receptor activation. Other drugs that alter transmitter synthesis can decrease synthesis and would cause decreased receptor activation. Drugs that affect transmitter production do not exert their effects on axonal conduction. The amount of transmitter produced does not directly affect transmitter storage. Some drugs that alter transmitter synthesis cause the synthesis of transmitter molecules that are more effective than the transmitter itself.)

A patient who has occasional migraine headaches tells a nurse that the abortive medication works well, but she would like to do more to prevent the occurrence of these headaches. The nurse will suggest that the patient: a. ask the provider about an adjunct medication, such as prochlorperazine. b. discuss the use of prophylactic medications with the provider. c. keep a headache diary to help determine possible triggers. d. take the abortive medication regularly instead of PRN.

c. keep a headache diary to help determine possible triggers (Keeping a headache diary to try to identify triggers to migraines can be helpful when a patient is trying to prevent them, and is the first step in managing headaches. Prochlorperazine is an antiemetic and does not prevent or abort migraine headaches. Prophylactic medications are used when headaches are more frequent. To prevent medication overuse headache, abortive medications should not be used more than 1 to 2 days at a time.)

A nurse is discussing alcohol abuse with a group of nursing students. One student asks whether alcohol consumption has any beneficial effects. The nurse replies that, in moderate amounts, alcohol: a. helps people to sleep well. b. improves sexual responsiveness. c. may protect against dementia. d. prevents hypothermia.

c. may protect against dementia. (In moderate amounts, alcohol helps preserve cognitive function in the older adult and may protect against dementia. Alcohol disrupts sleep and alters sleep cycles, reducing total sleeping time and the quality of sleep. Alcohol lowers inhibitions but diminishes sexual responsiveness. Alcohol dilates cutaneous blood vessels, which actually promotes heat loss.)

A patient is taking a beta1-adrenergic drug to improve the stroke volume of the heart. The nurse caring for this patient knows that this drug acts by increasing: a. cardiac afterload. b. cardiac preload. c. myocardial contractility. d. venous return.

c. myocardial contractility. (Beta1-adrenergic agents help increase the heart's stroke volume by increasing myocardial contractility. Cardiac afterload is determined primarily by the degree of peripheral resistance caused by constriction of arterioles; increasing afterload would decrease stroke volume. Beta1- adrenergic agents do not affect afterload. Cardiac preload is the amount of stretch applied to the cardiac muscle before contraction and is determined by the amount of venous return. Beta1- adrenergic agents do not affect cardiac preload. Venous return is determined by the systemic filling pressure and auxiliary muscle pumps and is not affected by beta1-adrenergic agents.)

A patient receives topical atropine to facilitate an eye examination. The nurse will tell the patient to remain in a darkened room or to wear sunglasses for several hours until the effects of the medication wear off. This teaching is based on the nurse's knowledge that muscarinic antagonists cause: a. elevation of intraocular pressure. b. miosis and ciliary muscle contraction. c. paralysis of the iris sphincter. d. relaxation of ciliary muscles.

c. paralysis of the iris sphincter. (By blocking muscarinic receptors in the eye, atropine causes paralysis of the iris sphincter, which prevents constriction of the pupil; consequently, the eye cannot adapt to bright light. This also causes an elevation in intraocular pressure, which increases the risk of glaucoma. However, it is not an indication for wearing darkened glasses. Muscarinic agonists cause miosis; atropine causes mydriasis. The effect of relaxing ciliary muscles focuses the eye for far vision, causing blurred vision.)

An infant is receiving a medication that has a narrow therapeutic range. The nurse reviews the medication information and learns that the drug is excreted by the kidneys. When giving the medication, the nurse will assess the infant for: a. decreased effectiveness of the drug. b. shorter period of the drug's effects. c. signs of drug toxicity. d. unusual CNS effects.

c. signs of drug toxicity. (Renal drug excretion is lower in infants, so drugs that are eliminated primarily by renal excretion should be given in reduced doses or at longer intervals. Drugs with a narrow therapeutic range should be monitored closely for toxicity. This drug likely will have intensified effects and be present for a longer time. Nothing indicates that unusual CNS effects will occur because of this alteration in excretion.)

A patient is diagnosed with moderate vitamin B12 deficiency. The nurse reviews the laboratory work and notes that the plasma B12 is low; also, a Schilling test reveals B12 malabsorption. The provider orders oral cyanocobalamin 500 mcg/day. The nurse will contact the provider to: a. discuss IM dosing. b. request an order for folic acid. c. suggest an increased dose. d. suggest platelet transfusion therapy

c. suggest an increased dose. (Patients with vitamin B12 deficiency associated with B12 malabsorption need increased doses of oral cyanocobalamin of 1000 to 10,000 mcg/day. It is not necessary to give this drug intramuscularly. Folic acid is indicated when B12 deficiency is severe. Platelets are given when B12 deficiency is severe.)

A nurse is teaching a group of nursing students how the CNS adapts to psychotherapeutic medications. Which statement by a nursing student indicates a need for further teaching? a. "Adaptation can lead to tolerance of these drugs with prolonged use." b. "Adaptation helps explain how physical dependence occurs." c. "Adaptation often must occur before therapeutic effects develop." d. "Adaptation results in an increased sensitivity to side effects over time."

d. "Adaptation results in an increased sensitivity to side effects over time." (With adaptation of the central nervous system to prolonged exposure to CNS drugs, many adverse effects diminish and therapeutic effects remain. Adaptation helps explain how tolerance and physical dependence occur, as the brain adapts to the presence of the drug. Therapeutic effects can take several weeks to manifest, because they appear to work by initiating adaptive changes in the brain.)

Which statement by the nurse indicates an understanding as to why atropine is given in the preoperative environment? a. "Atropine induces sleep". b. "Atropine works as a pain reliever". c. "Atropine impairs memory" d. "Atropine decreases secretions".

d. "Atropine decreases secretions". (Atropine decreases secretions from the salivary glands, which decreases the risk of aspiration during surgery.)

A patient tells a nurse that she drinks wine for its cardioprotective effects. She wants to know how much is safe. The nurse is correct in telling her that: a. "A couple of glasses of wine 1 or 2 days a week will help prevent heart disease." b. "Abstaining from alcohol completely is the best way to prevent heart disease." c. "If you drink only on weekends, you will minimize your risk of developing heart disease." d. "Cardioprotective effects are greatest in people who live an unhealthy lifestyle."

d. "Cardioprotective effects are greatest in people who live an unhealthy lifestyle." (Moderate consumption, which for women is 1 drink a day or less, has been shown to reduce the incidence of ischemic stroke, coronary artery disease, myocardial infarction, and heart failure. The pattern determines protection as well; moderate consumption 3 or 4 days a week rather than 1 or 2 days is more protective. These effects are greatest in people who have an unhealthy lifestyle, who do not eat fruits and vegetables, and who smoke. Having a couple of glasses of wine exceeds the moderate amount for women. Moderate consumption, as just described, has better protective benefits than complete abstinence. Drinking on weekends does not fit the pattern of 3 or 4 days per week that has been shown to be protective.)

A nurse is preparing a patient who will stop taking lorazepam [Ativan] for anxiety and begin taking buspirone [Buspar]. Which statement by the patient indicates a need for further teaching? a. "I can drink alcohol when taking Buspar, but not grapefruit juice." b. "I may need to use a sedative medication if I experience insomnia." c. "I may not feel the effects of Buspar for a few weeks." d. "I should stop taking the Ativan when I start taking the Buspar."

d. "I should stop taking the Ativan when I start taking the Buspar." (Ativan should not be withdrawn quickly; it must be tapered to prevent withdrawal symptoms. Moreover, Buspar does not have immediate effects. Because no cross-dependence occurs with these two medications, they may be taken together while the benzodiazepine is tapered. Because Buspar does not have sedative effects, patients can consume alcohol without increasing sedation. Levels of Buspar can be increased by grapefruit juice, leading to drowsiness and a feeling of dysphoria. Buspar can cause nervousness and excitement and does not have sedative effects, so patients with insomnia must use a sedative. Buspar does not have immediate effects.)

A nurse is providing patient education about colesevelam [Welchol], a bile-acid sequestrant. Which statement made by the patient demonstrates a need for further teaching? a. "Colesevelam will reduce my levels of low-density lipoprotein." b. "Colesevelam will augment my statin drug therapy." c. "I will not have to worry about having as many drug interactions as I did when I took cholestyramine." d. "I will need to take supplements of fat-soluble vitamins."

d. "I will need to take supplements of fat-soluble vitamins." (Colesevelam [Welchol] does not reduce absorption of fat-soluble vitamins as do other bile-acid sequestrants, so supplements are not needed. Colesevelam reduces the LDL cholesterol level, which is one of its therapeutic uses. Colesevelam augments statin therapy. Colesevelam does not significantly interact with or reduce the absorption of statins, digoxin, warfarin, or most other drugs.)

A patient will begin using a transdermal preparation of a muscarinic antagonist for overactive bladder (OAB). The nurse teaches the patient what to do if side effects occur. Which statement by the patient indicates the need for further teaching? a. "I can use sugar-free gum for dry mouth." b. "I may need laxatives for constipation." c. "I should keep the site covered to prevent other people from getting the medicine." d. "I will take Benadryl for any itching caused by a local reaction to the patch."

d. "I will take Benadryl for any itching caused by a local reaction to the patch." (Benadryl is an antihistamine, and even though it is not classified as a muscarinic antagonist, it has anticholinergic effects. Giving it with a muscarinic antagonist greatly enhances these effects, so it should not be used. Muscarinic antagonists cause dry mouth, and patients should be taught to use sugar-free gum or candies to help with this. Muscarinic antagonists can cause constipation, and laxatives may be used. Medication applied topically can be transferred to others who come in contact with the skin, so the site should be covered.)

A nurse is teaching a patient who has been diagnosed with hypothyroidism about levothyroxine [Synthroid]. Which statement by the patient indicates a need for further teaching? a. "I should not take heartburn medication without consulting my provider." b. "I should report insomnia, tremors, and an increased heart rate to my provider." c. "If I take a multivitamin with iron, I should take it 4 hours after the Synthroid." d. "If I take calcium supplements, I may need to decrease my dose of Synthroid."

d. "If I take calcium supplements, I may need to decrease my dose of Synthroid."

A patient in the emergency department has severe chest pain. The nurse administers morphine intravenously. The patient asks the nurse why morphine is given. Which response by the nurse is correct? a. "Morphine helps by reducing anxiety and relieving pain." b. "Morphine helps by reducing pain and dissolving clots." c. "Morphine helps by relieving pain and lowering blood pressure." d. "Morphine helps by relieving pain and reducing the cardiac oxygen demand."

d. "Morphine helps by relieving pain and reducing the cardiac oxygen demand." (IV morphine is the treatment of choice for STEMI-associated pain. Besides relieving pain, it promotes vasodilation and reduces cardiac preload, which lowers the cardiac oxygen demand. It does not reduce anxiety, dissolve clots, or lower blood pressure.)

A nurse provides teaching to a patient who will begin taking a drug with a known risk of hepatotoxicity. Which statement by the patient indicates a need for further teaching? a. "I should avoid taking acetaminophen while taking this drug." b. "I will need periodic evaluation of aspartate aminotransferase and alanine aminotransferase levels." c. "If I experience nausea, vomiting, or abdominal pain, I should call my provider." d. "Routine testing and early detection of problems will prevent liver failure."

d. "Routine testing and early detection of problems will prevent liver failure." (Drug-induced liver injury can progress from undetectable to advanced between routine tests; therefore, routine testing does not always prevent liver failure. Patients taking known hepatotoxic drugs should avoid other drugs, such as acetaminophen, that can cause liver damage. Aspartate aminotransferase (AST) and alanine aminotransferase (ALT) are liver enzymes that are routinely monitored when a patient is taking hepatotoxic drugs. Nausea, vomiting, and abdominal pain are signs of liver injury and should be reported.)

A nursing student asks the nurse about receptor specificity of adrenergic agonist medications. What will the nurse say? a. "As the dosage of these medications increases, drugs in this class are more selective." b. "Dopamine is selective for dopamine receptors and has no effects on alpha or beta receptors." c. "Epinephrine is the most selective alpha-adrenergic agonist medication." d. "Specificity is relative and is dose dependent."

d. "Specificity is relative and is dose dependent." (Receptor specificity is relative, not absolute, and is dose dependent. At low doses, selectivity is maximal, and selectivity declines as the dose increases. Dopamine is the only drug in this class that acts on dopamine receptors, but it also has effects on alpha and beta receptors. Epinephrine is the least selective of this class.)

A patient who has obsessive-compulsive disorder (OCD) has been undergoing behavioral therapy but continues to exhibit symptoms that interfere with daily life. Which intervention will the nurse expect the provider to order for this patient? a. Alprazolam [Xanax] b. Buspirone [Buspar] c. Deep brain stimulation d. Fluoxetine [Paxil]

d. Fluoxetine [Paxil] (Patients with OCD usually respond optimally to a combination of an SSRI, such as fluoxetine, and behavioral therapy. Alprazolam and buspirone are used to treat GAD. Deep brain stimulation is used when other therapies fail to treat OCD.)

A nurse is teaching a group of students about how CNS drugs are developed. Which statement by a student indicates a need for further teaching? a. "Central nervous system drug development relies on observations of their effects on human behavior." b. "Studies of new central nervous system drugs in healthy subjects can produce paradoxical effects." c. "Our knowledge of the neurochemical and physiologic changes that underlie mental illness is incomplete." d. "These drugs are developed based on scientific knowledge of CNS transmitters and receptors."

d. "These drugs are developed based on scientific knowledge of CNS transmitters and receptors." (The deficiencies in knowledge about how CNS transmitters and receptors work make systematic development of CNS drugs difficult. Testing in healthy subjects often leads either to no effect or to paradoxical effects. Medical knowledge of the neurochemical and physiologic changes underlying mental illness is incomplete. The development of CNS drugs depends less on knowledge of how the CNS functions and how these drugs affect that process and more on how administering one of these agents leads to changes in behavior.)

A nursing student asks about drugs that interfere with the termination of transmitter action. Which statement by the nurse is correct? a. "Drugs act on this process by altering the diffusion of the transmitter away from the synaptic gap." b. "Drugs can interfere with termination by either increasing or decreasing reuptake of the transmitter." c. "Drugs in this category lead to decreased activation by the transmitter in the synapse." d. "These drugs reduce either reuptake or degradation of the transmitter, causing an increase in receptor activation."

d. "These drugs reduce either reuptake or degradation of the transmitter, causing an increase in receptor activation." (Drugs that interfere with termination of transmitter action do so by blocking transmitter reuptake or inhibiting transmitter degradation, resulting in increased receptor activation, because more of the transmitter remains available. Diffusion of the transmitter occurs naturally, but it is a slow process with little clinical significance. Drugs that alter this process cause a decrease in reuptake, not an increase. The effect of drugs that interfere with termination of transmitter action is increased activation.)

A patient has a free T4 level of 0.6 ng/dL and a free T3 of 220 pg/dL. The patient asks the nurse what these laboratory values mean. How will the nurse respond? a. "These laboratory values indicate that you may have Graves' disease." b. "These results suggest you may have hyperthyroidism." c. "We will need to obtain a total T4 and a total T3 to tell for sure." d. "We will need to obtain a TSH level to better evaluate your diagnosis."

d. "We will need to obtain a TSH level to better evaluate your diagnosis."

A patient who has been newly diagnosed with adrenal hormone deficiency will begin taking hydrocortisone. The nurse provides teaching for this patient. Which statement by the patient indicates understanding of the teaching? a. "I may take all of my daily dose in the morning or divide it in half and take it twice daily." b. "I will need to take this medication until my symptoms completely clear, and then I may stop." c. "Side effects are common with hydrocortisone, even with therapeutic doses." d. "When I am sick, I should take 3 times the normal dose for 3 days in a row."

d. "When I am sick, I should take 3 times the normal dose for 3 days in a row."

A nurse provides dietary counseling for a patient newly diagnosed with type 1 diabetes. Which instruction should be included? a. "You may eat any foods you want and cover the glucose increase with sliding scale, regular insulin." b. "Most of the calories you eat should be in the form of protein to promote fat breakdown and preserve muscle mass." c. "Your total caloric intake should not exceed 1800 calories in a 24-hour period." d. "You should use a carbohydrate counting approach to maintain glycemic control."

d. "You should use a carbohydrate counting approach to maintain glycemic control."

A prescriber has ordered methyldopa for a patient with hypertension. The nurse teaches the patient about drug actions, adverse effects, and the ongoing blood tests necessary with this drug. The nurse is correct to tell the patient what? a. "If you have a positive Coombs' test result, you will need to discontinue the medication, because this means you have hemolytic anemia." b. "Methyldopa can be used for its analgesic effects and for its hypertensive effects." c. "Xerostomia and orthostatic hypotension are serious side effects and indications for withdrawing the medication." d. "You will need to contact the provider and stop taking the medication if your eyes look yellow."

d. "You will need to contact the provider and stop taking the medication if your eyes look yellow." (Hepatotoxicity is a serious adverse effect of methyldopa and is an indication for withdrawal of the drug to prevent fatal hepatic necrosis. Jaundice is a sign of liver toxicity. Patients should undergo periodic liver function tests while taking the drug. Liver function usually improves when the drug is withdrawn. A positive Coombs' test result is not an indication for withdrawal of the drug in itself. About 5% of patients with a positive Coombs' test result develop hemolytic anemia; withdrawal of the drug is indicated for those patients. Methyldopa does not have analgesic effects. Xerostomia and orthostatic hypotension are known side effects of methyldopa but usually are not serious.)

A patient in the postanesthesia recovery unit received ketamine [Ketalar] for right open reduction internal fixation surgery. What drug would be beneficial as a premedication to help minimize adverse reactions? a. Methohexital [Brevital] b. Sevoflurane [Ultane] c. Atropine [Sal-Tropine] d. Diazepam [Valium]

d. Diazepam [Valium] (To minimize the effects of ketamine, the patient should be premedicated with diazepam or midazolam to reduce the risk of an adverse reaction. Methohexital would further sedate the patient. Sevoflurane would further complicate sedation and would not be indicated. Atropine would not reduce the risk of an adverse reaction.)

A patient with epistaxis and a history of hemophilia A is admitted to the unit and is scheduled for replacement therapy. The nurse should prepare to administer which medication? a. Tranexamic acid [Cyklokapron] b. Aminocaproic acid [Amicar] c. Desmopressin [Stimate] d. Factor VIII

d. Factor VIII (The cornerstone of treatment for hemophilia A is replacement therapy with factor VIII. Tranexamic acid and aminocaproic acid are antifibrinolytic agents that act primarily by preventing the formation of plasmin from its precursor. Desmopressin promotes the release of factor VIII from the vascular endothelium and has the advantage of being cheaper than factor VIII. Also, it can be administered by nasal spray or by IV infusion. However, repeated use of desmopressin can deplete stored factor VIII.)

A patient is to receive a beta agonist. Before administration of this medication, which assessment finding would most concern the nurse? a. Pulse oximetry reading of 88% b. Blood pressure of 100/60 mm Hg c. Respiratory rate of 28 breaths per minute d. Heart rate of 110 beats/min

d. Heart rate of 110 beats/min (A beta agonist dilates respiratory smooth muscle, but as a side effect, it can stimulate the heart. A heart rate of 110 beats/min is a concern, because this medication may further increase the already elevated heart rate. A pulse oximetry reading of 88% is a concern, but the medication causes bronchodilation and increased oxygenation; this should increase the pulse oximetry reading. A blood pressure of 100/60 mm Hg is on the low side, but this medication may actually cause an increase in blood pressure as a side effect; this should not concern the nurse before administration of the medication. A respiratory rate of 28 breaths per minute is elevated; however, this medication should increase oxygenation by bronchodilation, and the patient's respiratory rate should decrease once oxygenation has improved. Therefore, this should not concern the nurse.)

A patient diagnosed with STEMI is about to undergo a primary percutaneous coronary intervention (PCI). Which combination of pharmacotherapeutic agents will be given to augment this procedure? a. Beta blocker and nitroglycerin b. Abciximab and a fibrinolytic drug c. Angiotensin-converting enzyme (ACE) inhibitor and aspirin d. Heparin, aspirin, and clopidogrel

d. Heparin, aspirin, and clopidogrel (Patients undergoing a primary PCI should receive heparin intravenously combined with aspirin and either clopidogrel or prasugrel. Abciximab and fibrinolytic drugs are not indicated. Beta blockers and nitroglycerin do not prevent thromboses. ACE inhibitors do not prevent thromboses.)

A patient who is recovering from a STEMI 3 months prior is in the clinic for a follow-up evaluation. The patient is taking 81 mg of aspirin, a beta blocker, and an ACE inhibitor daily and uses nitroglycerine as needed for angina. The patient's BMI is 24.5 kg/m2 , and serum LDL is 150 mg/dL. The patient has a blood pressure of 135/80 mm Hg. What will the nurse expect the provider to order for this patient? a. An antihypertensive medication b. Counseling about a weight loss diet c. Discontinuing the ACE inhibitor d. High-dose statin therapy

d. High-dose statin therapy (To help prevent recurrence of MI in patients post-STEMI, a high-dose statin should be given to patients with elevated cholesterol. This patient's blood pressure and BMI are normal, so antihypertensives and a weight loss diet are not recommended. The three drugs should be continued indefinitely.)

A nurse is teaching a patient about a drug that induces P-glycoprotein. The nurse will explain that this drug may cause which effect on other drugs? a. Decreased absorption in the intestines b. Decreased elimination through the kidneys c. Increased brain exposure d. Increased fetal absorption

d. Increased fetal absorption (Drugs that induce PGP can increase drug export from cells of the intestinal epithelium into the intestinal lumen, thus decreasing absorption of the drug. PGP inducers also increase drug elimination and decrease brain and fetal drug exposure.)

A patient who is taking morphine for pain asks the nurse how a pain medication can also cause constipation. What does the nurse know about morphine? a. It binds to different types of receptors in the body. b. It can cause constipation in toxic doses. c. It causes only one type of response, and the constipation is coincidental. d. It is selective to receptors that regulate more than one body process.

d. It is selective to receptors that regulate more than one body process. (Morphine is a medication that is selective to receptor type that regulates more than one process. Because it is selective to receptor type, it does not bind to different types of receptors. Constipation is a normal side effect and is not significant for toxicity.)

A patient with Cushing's syndrome has undergone surgery and radiation treatment. The nurse will expect to teach the patient about which medication? a. Cosyntropin b. Dexamethasone c. Fludrocortisone acetate [Florinef] d. Ketoconazole [Nizoral]

d. Ketoconazole [Nizoral]

A patient with type 1 diabetes is eating breakfast at 7:30 AM. Blood sugars are on a sliding scale and are ordered before a meal and at bedtime. The patient's blood sugar level is 317 mg/dL. Which formulation of insulin should the nurse prepare to administer? a. No insulin should be administered. b. NPH c. 70/30 mix d. Lispro [Humalog]

d. Lispro [Humalog]

Which statement is accurate about the long-term complications of diabetes? a. Long-term complications are almost always the result of hypoglycemia and ketoacidosis. b. The complication rates for patients with optimally controlled type 2 diabetes are the same as for those whose disease is not optimally controlled. c. Optimal control of type 1 diabetes produces excessive episodes of life-threatening hypoglycemia. d. Optimal control of both types of diabetes reduces the risk of eye, kidney, and nerve damage.

d. Optimal control of both types of diabetes reduces the risk of eye, kidney, and nerve damage.

The nurse is teaching a patient how a medication works to treat an illness. To do this, the nurse will rely on knowledge of which topic? a. Clinical pharmacology b. Drug efficacy c. Pharmacokinetics d. Pharmacotherapeutics

d. Pharmacotherapeutics (Pharmacotherapeutics is the study of the use of drugs to diagnose, treat, and prevent conditions. Clinical pharmacology is concerned with all aspects of drug-human interactions. Drug efficacy measures the extent to which a given drug causes an intended effect. Pharmacokinetics is the study of the impact of the body on a drug.)

The nurse is teaching a patient about taking warfarin and asks if the patient takes aspirin. This assessment by the nurse reflects a knowledge of which type of drug interaction? a. Creation of unique effects b. Increased therapeutic effects c. Inhibitory effects d. Potentiative effects

d. Potentiative effects (A potentiative effect is one in which one drug intensifies the effects of another. Both warfarin and aspirin suppress blood clotting, and the combination may increase the risk of bleeding, which is an intensified adverse effect. Creation of a unique effect is a rare occurrence in which the combination of two drugs creates a response not seen with either drug when given alone. Increased therapeutic effects are a type of potentiative effect; however, in this case the combination of two drugs would increase the desired effects. An inhibitory effect is a type of pharmacodynamic effect that occurs when an antagonist drug inhibits the action of an agonist drug at the same receptor site.)

A patient asks a nurse why drugs that have been approved by the FDA still have unknown side effects. What will the nurse tell the patient? a. Testing for all side effects of a medication would be prohibitively expensive. b. Patients in drug trials often are biased by their preconceptions of a drug's benefits. c. Researchers tend to conduct studies that will prove the benefits of their new drugs. d. Subjects in drug trials do not always represent the full spectrum of possible patients.

d. Subjects in drug trials do not always represent the full spectrum of possible patients. (All drug trials are limited by a relatively small group of subjects who may not have all the characteristics of people who will be using the drug; therefore, some side effects go undetected until the drug is in use. Although drug trials are very expensive, this is only an indirect reason that they do not detect all side effects before approval. In theory, well-designed drug trials, using blinded studies, minimize or eliminate subject bias. Designing studies to prove desired results is unethical.)

An older adult patient is diagnosed with hypothyroidism. The initial free T4 level is 0.5 mg/dL, and the TSH level is 8 microunits/mL. The prescriber orders levothyroxine [Levothroid] 100 mcg/day PO. What will the nurse do? a. Administer the medication as ordered. b. Contact the provider to discuss giving the levothyroxine IV. c. Request an order to give desiccated thyroid (Armour Thyroid). d. Suggest that the provider lower the dose..

d. Suggest that the provider lower the dose..

A nurse caring for a 5-year-old child notes that the child has discoloration of several teeth. When taking a medication history, the nurse will ask about which group of medications? a. Glucocorticoids b. Salicylates c. Sulfonamides d. Tetracyclines

d. Tetracyclines (Tetracyclines cause discoloration in developing teeth in children. Glucocorticoids are associated with growth suppression. Salicylates are associated with Reye's syndrome. Sulfonamides are associated with kernicterus in newborns.)

A patient with type 1 diabetes who takes insulin reports taking propranolol for hypertension. Why is the nurse concerned? a. The beta blocker can cause insulin resistance. b. Using the two agents together increases the risk of ketoacidosis. c. Propranolol increases insulin requirements because of receptor blocking. d. The beta blocker can mask the symptoms of hypoglycemia.

d. The beta blocker can mask the symptoms of hypoglycemia.

A prescriber orders transdermal clonidine [Catapres TTS] for a patient with hypertension. What will the nurse teach this patient? a. That medication given by transdermal patch has fewer systemic side effects b. That localized skin reactions are uncommon c. To apply the patch to intact skin on the forearm or leg d. To change the patch every week

d. To change the patch every week (Transdermal patches are to be changed every 7 days. Medication administered by patch has the same therapeutic effect and adverse effects as that given by other routes, except that localized skin reactions may occur and are common with clonidine patches. The patch should be applied to intact, hairless skin on the upper arm or torso.)

A patient who has a history of asthma experiences three or four migraine headaches each month. The patient uses sumatriptan [Imitrex] as an abortive medication and has developed medication overuse headaches. The patient asks the nurse what can be done to prevent migraines. The nurse will suggest that the patient discuss which preventive medication with the provider? a. Botulinum toxin b. Meperidine [Demerol] c. Timolol d. Topiramate [Topamax]

d. Topiramate [Topamax] (Topiramate can be used for migraine prophylaxis, and its benefits appear equal to those of the first-line beta blockers. Botulinum toxin can be used for migraine prophylaxis in patients who have 15 or more headaches a month. Meperidine may be used as abortive therapy but has addictive potential. Timolol is a beta blocker; this patient has asthma, and because beta blockers cause bronchoconstriction, these agents are not recommended.)

A patient is in the intensive care unit after a myocardial infarction. The nurse notes that the QT interval on this patient's electrocardiogram has been elongating. The nurse is concerned that which cardiac dysrhythmia may occur? a. AV block b. Bradycardia c. Supraventricular tachycardia d. Torsades de pointes

d. Torsades de pointes (Torsades de pointes is a dysrhythmia that can occur with prolongation of the QT interval and can progress to fatal ventricular fibrillation. A prolonged QT interval does not signal the development of AV block, bradycardia, or SVT.)

A patient with hemophilia B is admitted to the hospital for surgery. The patient's history reveals long-term use of factor replacement as prophylaxis for bleeding episodes. The nurse anticipates that the provider will order which medication to prevent excessive perioperative bleeding? a. Desmopressin b. Factor VIII c. Ibuprofen d. Tranexamic acid

d. Tranexamic acid (Antifibrinolytic drugs, such as tranexamic acid, which prevent the breakdown of fibrin, are used as adjuncts to replacement factor in special cases in which bleeding is likely. Desmopressin is used in patients with mild hemophilia A to help control bleeding episodes. Hemophilia B is a deficiency of factor IX, so giving factor VIII would not be appropriate. Ibuprofen would only reduce clotting and should not be used.)

A nurse caring for a patient after a cesarean section learns that the patient received isoflurane [Forane] during the surgery. What will the nurse monitor most closely in this patient? a. Hypertension b. Nausea and vomiting c. Constipation d. Uterine tone

d. Uterine tone (Although having only moderate effects on skeletal muscles, isoflurane does promote relaxation of uterine smooth muscle; when used in obstetrics, isoflurane may delay delivery and possibly increase postpartum bleeding, so the nurse should monitor uterine tone. Isoflurane may cause hypotension. Nausea, vomiting, and constipation are not concerns.)

A patient is admitted to the hospital. The patient's initial laboratory results reveal megaloblastic anemia. The patient complains of tingling of the hands and appears confused. The nurse suspects what in this patient? a. Celiac disease b. Folic acid deficiency c. Iron deficiency anemia d. Vitamin B12 deficiency

d. Vitamin B12 deficiency (When patients present with megaloblastic anemia, it is essential to distinguish between folic acid deficiency and vitamin B12 deficiency. If neurologic deficits are observed, vitamin B12 deficiency is more likely to be the cause. This patient does not have signs of celiac disease. Iron deficiency anemia would be indicated by a low hemoglobin and hematocrit.)

The nurse is caring for a patient receiving desmopressin [Stimate]. The nurse is performing a physical assessment and notes that the patient is drowsy, listless, and complains of a recent onset of headache. Which finding would be most consistent with these symptoms? a. Hyperglycemia b. Hypernatremia c. Hypertension d. Water intoxication

d. Water intoxication

A patient was given a 30-day supply of Feosol and has been taking the drug for 4 weeks for iron deficiency anemia. The patient's initial hemoglobin was 8.9 gm/dL. The nurse notes that the hemoglobin has risen to 9.7 gm/dL. What will the nurse ask the patient about? a. Dietary iron intake b. Gastrointestinal (GI) upset c. Whether stools have been tarry or black d. Whether the prescription needs to be refilled

d. Whether the prescription needs to be refilled (When therapy is successful, the hemoglobin level increases by 2 gm/dL within 1 month. If the hemoglobin does not increase as expected, patients should be asked about compliance. If a patient reports that the prescription does not need to be refilled, the medication probably has not been taken as prescribed. Dietary iron intake is not a part of iron replacement therapy but is an important part of the prevention of anemia. GI upset and tarry, black stools are expected side effects of iron products.)

The nurse is caring for a child who has ingested a toxic amount of aspirin. The provider orders an intravenous drug that will increase pH in the blood and urine. The nurse understands that this effect is necessary to: a. decrease the gastric absorption of aspirin. b. decrease the lipid solubility of aspirin. c. increase the serum protein binding of aspirin. d. increase the urinary excretion of aspirin.

d. increase the urinary excretion of aspirin. (The phenomenon of pH-dependent ionization can be used to accelerate renal excretion of drugs. When children have been exposed to toxic amounts of aspirin, they can be treated, in part, by giving an agent that elevates urinary pH, leading to less passive reabsorption of the now ionized molecules of aspirin and, hence, more excretion. Elevating the pH of the blood and urine does not affect absorption in the stomach. Ionization of aspirin does not affect lipid solubility or protein binding.)

A child with severe hemophilia A receives factor VIII. The nurse expects this drug to be given: a. on demand when bleeding occurs. b. on demand when plasma factor VIII activity is less than 1% above normal. c. prophylactically once weekly to maintain normal factor VIII activity. d. prophylactically 3 times weekly to maintain factor VIII activity above 1% of normal.

d. prophylactically 3 times weekly to maintain factor VIII activity above 1% of normal. (Children with severe hemophilia are primary candidates for prophylactic therapy with factor VIII, and this is given every other day or 3 times weekly with a goal of maintaining factor VIII activity above 1% of normal. On-demand therapy is indicated when patients are bleeding or are about to undergo surgery.)

. A patient arrives in the emergency department with a heart rate of 128 beats/min and a temperature of 105°F. The patient's skin feels hot and moist. The free T4 level is 4 ng/dL, the free T3 level is 685 pg/dL, and the TSH level is 0.1 microunits/mL. The nurse caring for this patient will expect to administer: a. intravenous levothyroxine. b. iodine-131 ( 131I). c. methimazole [Tapazole]. d. propylthiouracil (PTU) d. propylthiouracil (PTU).

d. propylthiouracil (PTU).

A patient is brought to the emergency department after a motor vehicle accident. The patient's speech is slurred. The nurse notes the smell of alcohol on the patient's breath and observes hand tremors. The patient's blood alcohol level is 0.4%. The nurse will expect to: a. find that the patient has lost consciousness within a short time. b. administer naltrexone [ReVia] and prepare for gastric lavage. c. give carbamazepine to reduce the risk of seizures. d. provide mechanical ventilation and oxygen.

d. provide mechanical ventilation and oxygen. (A blood alcohol level that exceeds 0.4% poses a substantial risk of respiratory depression. Patients who are chronic abusers of alcohol may develop tolerance to other effects of increased blood levels, such as sedation, or behavioral changes, but there is very little tolerance to respiratory depression. A patient with a blood alcohol level of 0.4% must be treated for respiratory depression, usually with mechanical ventilation. If this patient has developed tolerance, which is likely because loss of consciousness has not already occurred, the nurse cannot expect that the patient will lose consciousness. Naltrexone is not used for acute toxicity. Carbamazepine is used as an adjunct to benzodiazepines and may be used after this patient's immediate needs have been addressed.)

A patient with antidiuretic hormone deficiency is receiving desmopressin (DDAVP). The nurse will teach this patient to: a. avoid grapefruit juice. b. increase sodium intake. c. monitor blood pressure. d. reduce fluid intake.

d. reduce fluid intake.

A patient who is experiencing alcohol withdrawal is given a benzodiazepine. The nurse understands that this drug is effective because: a. the alcohol does not interact with the benzodiazepine. b. the benzodiazepine potentiates alcohol withdrawal symptoms. c. the benzodiazepine relieves muscle spasms and spasticity. d. the patient has a cross-dependence to the benzodiazepine.

d. the patient has a cross-dependence to the benzodiazepine. (Benzodiazepines are given to ease withdrawal from alcohol because of cross-dependence with these drugs and alcohol, enabling the benzodiazepine to suppress withdrawal symptoms. Alcohol and benzodiazepines can potentiate one another. The benzodiazepine does not potentiate withdrawal symptoms. Benzodiazepines relieve muscle spasms, but this is not why they are given for alcohol withdrawal.)

A 7-year-old child who is otherwise healthy is receiving mecasermin [Increlex] replacement therapy to treat severe primary deficiency of insulin-like growth factor-1 (IGF-1). The child develops tonsillar hypertrophy. The nurse anticipates that the provider will recommend: a. antibiotics. b. reducing the dose of mecasermin. c. discontinuing the mecasermin. d. tonsillectomy.

d. tonsillectomy.


Related study sets

Module 8: Pure Competition in the Short Run

View Set

MKT-378-001 Test #3 JSU (Dr. Coco)

View Set

chapter 22 the short-run trade-off between inflation and unemployment

View Set

Chapter 7: Political Participation: Activating the Popular Will

View Set